Categories
Chicago Fields Suggested Reading Undergraduate

Chicago. Recommended public finance textbooks. Viner’s list, 1924

 

The original memo sent to Jacob Viner asking for the names of a few textbooks suitable for college class in the field of public finance is a carbon copy of a common memo, except for the name “Mr. Jacob Viner” and field “Public Finance” that are both clearly typed onto the carbon copy. It appears that the chairman L. C. Marshall might have been surveying his Chicago colleagues to assemble a list of college textbooks by field. There might be other such inquiries with responses, but judging from where I found this memo to Viner, one would have to plow through the Chicago economic department records where the memos are filed by recipients. I’ll keep my eyes open.

The first textbook listed by Viner was written by the 1926 Chicago Ph.D., Jens Peter Jensen, whose dissertation was on the general property tax.

Obituary:  In Memoriam: Jens P. Jensen, 1883-1942 by John Ise in The American Journal of Economics and Sociology, Vol. 2, No. 3 (Apr., 1943), pp. 391-392.

____________________

From the Preface of Jens P. Jensen’s (Department of Economics, University of Kansas) Problems of Public Finance, p. ix.

“Professors Roy G. Blakey of the University of Minnesota and H. A. Millis of the University of Chicago were my teachers in public finance, and through them my interest in the field was aroused and quickened. Dr. J. Viner of the University of Chicago has carefully read the manuscript and suggested many redeeming changes.”

____________________

The University of Chicago
The School of Commerce and Administration

Memorandum to Mr. Jacob Viner from L.C. Marshall
October 2, 1924

Will you please jot down on this sheet the names of two or three texts suitable for college class use in the field of Public Finance?

LCM:OU

*  *  *  *  *  *

Viner’s reply

Jens [Peter] Jensen. Problems of Public Finance.  Crowell [1924]

C. J. Bullock. Selected Readings in P. F. Ginn & Co. [2nded., 1920]

W. M. Daniels, Elements of Public Finance [including the Monetary System of the United States]. Holt & Co. [1899]

H. L. Lutz has a good text in press [D. Appleton and Company, 1924;  fourth edition, 1947]

J.V.

Source: University of Chicago Archives. Department of Economics. Records. Box 35, Folder 14.

Image Source: Jacob Viner (facing camera) playing bridge with Mr. Grabo, Mr. Prescott, and Ralph Sanger, instructor of Mathematics. University of Chicago Photographic Archive, apf1-08487, Special Collections Research Center, University of Chicago Library.

Categories
Curriculum Economics Programs Fields Harvard Statistics

Harvard. Report on statistics and national income courses. Crum and Frickey, 1945

 

William Leonard Crum and Edward Frickey taught Harvard’s economic statistics courses in the 1930s and 1940s.  Paul Samuelson recounted his second semester (Spring 1936) as a graduate student following his previous semester’s worth of Crum: “…I was able to learn genuine modern statistics from E. B. Wilson, bypassing Edwin Frickey (who with Leonard Crum taught at Harvard courses against modern statistics!)” [On this, Roger E. Backhouse’s Vol I: Becoming Samuelson, 1915-1948, p.101].

Reading the following intradepartmental report on economic statistics courses and how to integrate national income and product accounting into the graduate curriculum that was written by a committee of two (Crum and Frickey), one discovers that even a decade after Samuelson’s experience, the proper preparation of “ink charts” was a subject that warranted faculty discussion.  Harvard Ph.D. Robert Solow later went to Columbia to play catch-up ball with respect to statistical analysis before starting his M.I.T. contract.  Harvard economics was a full generation behind the times with respect to statistical method at mid-20th century.

A 1947 Crum/Frickey  joint memo regarding preparation for taking the comprehensive field exam in statistics has been posted earlier.

______________________

6 March 1945

Report on the course offerings in Statistics, and in National Income

At the Department meeting of 13 February, 1945, the undersigned were named a committee to study course offerings and proposed offerings in Statistics and in National Income, discuss their findings with the Chairman, and report to the Department. Attached are the two reports: I, on Statistics; II, (page 10) on National Income.

W.L. Crum
Edwin Frickey

*  *  *  *  *  *  *  *  *  *  *  *  *

I. The Offering in Statistics

At the meeting of the Harvard Economics Department on 13 February, 1945. W. L. C. and E. F. sought opinions from colleagues as to additional instruction needed in statistics and as to changes needed in existing instruction. The following is in part a report of the informal discussion, in part an indication of what W. L. C. and E. F. think can advisedly be done. The present statement is preliminary; a more definitive report will be prepared, after consultation with H. H. B., for submission to the Department at a later meeting.

  1. Opinion was expressed that many of our graduate students show conspicuous lack of ability to present statistical material in the form of chart or table, for example, in theses. Instruction in statistics here has for several years relied upon capacity of students to learn by emulation—they have abundant opportunity to acquaint themselves with good statistical presentation, both tabular and graphic, in our courses in statistics and in the source materials of other courses. In course 21a, some instruction is incidentally given in orderly tabulation of limited sorts, but we make no attempt to teach students to prepare ink charts. Apparently, something more is needed; and three suggestions, perhaps all to be followed together, are made:
    1. By compressing some other parts of the work, we can include a small amount of instruction on presentation in course 21a. This should help put those graduate students who are required to take that course here on the right track.
    2. For students not required to take 21a, because they have had the “equivalent” elsewhere, one possibly helpful device is to require in course 121b a written report involving presentation in tabular and graphic form. Such report could be graded if sufficient funds are available to cover the grading, and the instructors could make a moderate effort to advise particular students about defects in their reports. The reports would presumably be required of all students in 121b, whether or not they had had 21a.
    3. The Department’s specialists in statistics could advise any graduate student, whose thesis involved matters of statistical presentation, concerning such matters. When the Department acquires a general research laboratory, with a regular supervisor, the supervisor could give such advice. In the meantime, the instructors in statistics could stand ready to give such advice in appropriate cases. The undersigned emphasize that this advice should be understood to concern presentation of statistical materials: they do not feel but they should be called upon ordinarily to advise such as student about sources of statistics for his thesis, or about the methods of analyzing the statistics, or about their interpretation. They have often given advice on such matters in certain cases, and will continue to do so, but take the stand that they should not be regarded as under the obligation to give such advice to all comers. The point is that; if the candidate proposes to write a statistical thesis in any field of economics, a vital part of his job is to obtain, analyze, and interpret his data. We see no reason why faculty specialists in statistics should make an extraordinary contribution to a thesis which happens to have quantitative aspects.
  2. Little emphasis appeared, in any opinions expressed, on the need for laboratory instruction in statistics in our graduate offering. Some suggestion was advanced that the “homework” type of problem task could helpfully be employed. W. L. C. and E. F. have a little faith that much could be accomplished in this way – the great advantage of the supervised laboratory is that the supervisor can get students actively started on the task and can catch and clear away difficulties as they arise. (We assume, of course, any problem work of this sort, in graduate courses, should be on an advanced – not elementary – level.) To meet this suggestion, we propose only that point A2 above be put into effect, and that the following change in present operations be considered. At present, course 121a includes two home-work problems, which stretch over several weeks, but are not graded and are not used as bases for specific advice to individual students. The proposed change is that these problems be handed in, and treated like the problem described in A2. (In these cases, as in that case, grading of the reports would be feasible if funds are available for the purpose.)

An emphatic suggestion was made that graduate students have the use of laboratory equipment, and be made welcome in the laboratory. We do not believe this can be managed with the laboratory facilities of course 21a. We note, however, that a moderate chance now exists that the University will presently provide the Department with a research laboratory in statistics, adequately equipped, and under competent supervision. If and when this is done, no difficulty will arise in making ample place for work by graduate students on any statistical tasks in which they may properly be interested. We remark that the arguments in favor of a general research laboratory in statistics are much more likely to bring conviction in responsible quarters that the argument, however strongly put, in favor of facilities merely for the occasional use of graduate students.

  1. Supposing we are to give an additional half graduate course in statistics, opinions pointed toward three alternatives:
    1. A course in theory, intermediate between course 121a and Prof. Wilson’s course 122b. This does not appear a good use of our manpower, for the election in such a course would inevitably be small, especially as the mathematics prerequisites would necessarily be much more severe than those – almost nil – on which we now limp through 121a.
    2. A further course was suggested – beyond 121b and perhaps alternating with it – in topics in the application of statistics to economic fields. Economics 121b now includes a selected list of such topics, which varies moderately from year to year; but it is by no means a comprehensive coverage of all even of the major possibilities. We could readily prepare an additional half course to be called 121c of further topics in the applied fields, and many students would probably like such a course. Such a course can be described as follows:

Economics 121c will be a half-course which might be entitled Topics in Applied Economic Statistics. Economics 21a or its equivalent will be a prerequisite. Properly qualified undergraduates may, with the consent of the instructor, be admitted to economics 121c.

Economics 121c will deal with statistical problems arising in connection with the use of basic statistical data in a selected list of economic topics. (As compared with 121b this course will lay more emphasis on the basic material and less emphasis on statistical theory.)

On each topic each student will be expected to familiarize himself with the immediate and the basic sources of the main materials, through actual examination of such materials, and to present a critical appraisal of these fundamental statistics. The instructor will give a succinct historical background – an outline of the principal work which is already been done on the topic. The instructor and the class will work out together conclusions as to what are the leading issues involved, and will consider what it is that statisticians are trying to measure and what they should be trying to measure.

Such topics as the following will be included:

Consumption
Commodity prices
Cost-of-living
Employment and unemployment
Wages
Money and Banking
Production and Trade (certain phases)
Balance of international payments
Public Finance

        1. The subordinate suggestion that, in this case, basic preparation for the oral exam and also the write-off field might consist of 121a and either 121b or 121c, was advanced. A strong objection to this appears in the fact that 121b, although made up largely of topics in applied statistics, now includes – and should continue to do so – certain topics which need to be covered by every general economist (we do not here have in mind the statistical specialist) who is to have “literacy” in the field of economic statistics today. Several of the “applied” topics now in 121b include in fact fundamental matters of statistical theory needed by all economists, and not elsewhere covered in our instruction. These include, for example: the theory of index numbers, statistical deflation, secular trends in business cycles, the basic theory of measuring production and income, and at least demand and cost curves not to mention more sophisticated matters of econometrics. These essentially theoretical topics in statistics should remain part of the basic graduate year course in statistics. (This goes also for our present topic of national income: even if the Department offer a course in that subject, the course will not be taken by all students, and all should have at least the brief survey now in 121b). For the foregoing reason, we emphatically urge that 121a and 121b stand as the basic year course in the field, and that the new course 121c be regarded as an additional – but not an alternative–half course.
        2. The subordinate suggestion at 121b and 121c be given in alternate years appears to fall for the same reason given in C2a.

 

    1. Instead of the course described under C2 suggestion was made that we introduce a course in administrative (we use this word provisionally, for want of a better) statistics – mainly, but not exclusively, governmental statistics. We have not outlined such a course in full, but can suggest its nature by indicating that it would emphasize the problems encountered in actually doing statistical work in government or private agencies. Such topics as the preparation and use of index numbers of prices and production; the compilation and use of data on employment and the labor force; statistics of farm production and operation; the gathering of and analysis of facts concerning trade, both foreign and domestic; financial data such as are developed by the treasury, the S. E. C, the F. R. B., and private agencies; statistics used in the analysis of particular enterprises; the rapidly developing field of quality control in industry, suggest themselves for inclusion. The nature of the course can also be indicated by somewhat loose contrast with the course described under C2 above: in that course, the point of view is of the user (economist, or other analyst) of statistics, and attention is given to the origin of the statistics only in so far as it is needed to guide and inform the user. In this course, the point of view is of the maker of statistics, and attention is given to the use of the statistics only in so far as it is needed to guide the maker in his work. This course would go far toward meeting the contention that our students, while well founded in statistical theory, are not ready to handle the kind of statistical tasks which they encounter in government or other research agencies.

At the moment we are not ready to choose between the courses described under C2 and C3, the former (and obviously the latter) being understood as in addition to, and not alternative to, 121b.

 

  1. No opinion was expressed concerning course 122b, and we think it should continue to be given in alternate years.

No opinion was offered concerning the content of course 121a. We have in mind some compression of one of the topics know given. This, plus the longer term under the peace-time schedule, will enable us to give more satisfactory attention to the topic of small samples.

We were commissioned to report also on national income. This is covered in a separate memorandum.

 

  1. We layout now, in tentative form and subject to revision by the Department, our recommendation as to the entire offering in statistics in the early post-war years.

21a. Substantially as at present, but with the change outlined in A1.

121a. Substantially as at present, but with the change outlined in B and the change noted in D.

121b. Substantially as at present, but with the change outlined in A2.

(Courses 121a and 121b to be regarded as the core of the preparation in the field of statistics, and to be recommended to the candidates for the general oral in statistics as the most helpful unit in their preparation.)

121c. A new half course, either that described under C2 or that under C3. To be open to graduate students who have had 21a or by consent of the instructor to those who have had the equivalent of 21a, and by consent of the instructor to properly qualified undergraduates who have had 21a.

122b. Substantially as at present, and to be given in alternate years as at present.

*  *  *  *  *  *  *  *  *  *  *  *  *

II. Offering in National Income

The suggestion is made that a half course, at the graduate level, in National Income be offered. The main purpose of such a course would be to give our students an extensive factual basis for their use of national income concepts and data in a wide range of our theoretical and applied fields. The course by itself could provide only a beginning for specialization the subject of national income for its own sake, and we do not understand that the Department contemplates recognizing the field in that subject.

While the course should be concerned primarily with the facts of national income, we understand that some attention could properly be given to the interpretation of those facts into their economic and social implications. Moreover, even to handle properly the factual side, the course would need give much attention to matters of definition and concept, matters which actually stand at the root of most of the “problems” of measuring national income and its chief constituents.

The core of the course would consist of the presentation, discussion, and criticism of the existing statistical facts on the national income and its constituents. These materials would presumably be limited to the United States; although some of the critical portions of the course, dealing with concepts and the like, would necessarily make large drafts on studies in certain other countries. Emphasis would be on the problems of measurement, the effectiveness and validity of the methods used, and the appropriateness of the results obtained as answers to questions posed by the economist.

In addition to the over-all aggregate of national income, viewed in real and money terms and in its variations over time, the course would examine the chief constituents of national income. These would include:

  1. Contributions to national income by various types of economic activity.
  2. Contributions from various geographical regions (much less is known on this.)
  3. Allocation, so far as it is known, to the several factors of production.
  4. Distribution according to size of income (money income) received by individuals.
  5. Distribution of income according to use: consumption expenditures of individuals (perishable, semi-durable), consumption through government, savings (by individuals, by enterprises, by government).
  6. Capital formation, and its relation to savings.
  7. Relation of taxes and public expenditures to the flow of income.

Your committee makes no recommendation as to the personnel to be assigned the task of conducting such a course. It does recommend: that the course be limited to graduate students, and to those advanced concentrators who receive permission from the instructor(s); that all students who take the course be required to have completed one half year course at the graduate level in economic theory and in statistics; that the course be given each year, rather than in alternate years; that the course be considered as a pro-seminar in statistics for the purpose of excuse – under our existing rules for reducing the oral examination to three fields – from the oral examination in statistics.

 

Source: Harvard University Archives. Department of Economics, Correspondence & Papers, 1902-1950. Box 23. Folder “Course Announcement 1945-46”.

Image Source: Crum and Frickey in the Harvard Class Album, 1942 and 1950.

Categories
Curriculum Fields Harvard

Harvard. Mathematical Economics Recognized as Subfield of Theory. E.B. Wilson, Crum, and Schumpeter, 1933

 

What I find particularly striking in the following report of the Committee on Instruction in Mathematical Economics at Harvard (note the  first named of the trio is E. B. Wilson) is the forecast that economics graduate students will need to acquire tools of mathematical economics and statistics already in the mid 1930s because they will need them later, 1953-63, when they will be “at the height of their activity” and by which time (implicitly) the “rapidly increasing importance of theoretical and statistical work involving higher mathematics” will have caught up with them. I have appended the course names for the statistics and mathematics courses referred to by number in the report.

Related postings: 

_____________________

Meeting of the Committee (Wilson, Crum, Schumpeter) on
Instruction in the Mathematical Economics
Tuesday, May 9 [1933]

In view of the rapidly increasing importance of theoretical and statistical work involving higher mathematics, and of the possibility that a considerable number of economists may have to be adequately familiar with both mathematical theory and statistical procedure twenty to thirty years from now, that is, when many of our present students will be at the height of their activity, the Committee (Wilson, Crum, Schumpeter) agreed on the following recommendations to be submitted to the Department which they believe to be both necessary and sufficient in order to provide facilities for events to work in mathematical theory as applied to economics:

(1) Any student who may wish to do so should be allowed to offer mathematical economics as his special field within the requirements for the Ph.D. This would involve but a slight alteration of existing practice which permits students to choose some branch of economic theory as a special field. The committee’s suggestion is merely that mathematical economics should be added to the other special subjects in economic theory which a student may select.

It seems desirable, moreover, to permit that any such student may select mathematics or rather some branch of pure or applied mathematics in place of one of the two remaining fields he has to offer.

(2) Advanced work in mathematical economics should conform to modern tendencies by stressing equally the mathematical side of economic theory and mathematical statistics. No student who elects mathematical economics as his special field should be allowed to do the one without the other. Especially courses 31a and 32b should be required also from students mainly interested in pure theory.

(3) Work in the Department of Mathematics through Math 5 should be considered as the minimum requirement as to mathematical training. Credit should be given only for Math 5, but not for any of the still more elementary course preparatory to it, which most of the students taking up mathematical economics will have had anyhow in their undergraduate period.

(4) No further steps should be taken at present. It seems best to see what the response will be before attempting to organize a special graduate course. The mathematical aspect of our subject is being dealt with in some courses already, and any Ph.D. candidates who may present themselves in case the rules be altered as recommended could easily be taken care of individually.

*  *  *  *  *  *  *  *

Copy of Letter from Harold H. Burbank to Joseph Schumpeter

October 3, 1933

Dear Joe,

I have read and approved without qualification the report of the Committee on Instruction in Mathematical Economics.

I think this report should be brought before the Department on the evening of Tuesday, October 10.

Very sincerely yours,

Prof. J. A. Schumpeter
2 Scott Street

HHB:VS

*  *  *  *  *  *  *  *

Graduate Instruction in the Mathematical Economics
Department Vote, October 10, 1933

In view of the rapidly increasing importance of theoretical and statistical work involving higher mathematics, and of the possibility that a considerable number of economists may have to be adequately familiar with both mathematical theory and statistical procedure twenty to thirty years from now, that is, when many of our present students will be at the height of their activity, the Committee (Wilson, Crum, Schumpeter) agreed on the following recommendations to be submitted to the Department which they believe to be both necessary and sufficient in order to provide facilities for events to work in mathematical theory as applied to economics.

The Department voted to accept the recommendations stated as follows:

(1) Any student who may wish to do so should be allowed to offer mathematical economics as his special field within the requirements for the Ph.D. This would involve no alteration of existing practice, which permits students to choose some branch of economic theory as a special field. The committee’s suggestion is that mathematical economics should be admissible.

(2) Any students using mathematical economics as his special field should be allowed to offer some branch of pure or applied mathematics as an allied field.

Work in the Department of Mathematics through Math 5, or the equivalent, should be considered as the minimum requirement as to mathematical training. Credit should be given only for Math 5, but not for any more elementary course preparatory to it.

(3) Advanced work in mathematical economics should conform to modern tendencies by stressing equally the mathematical side of economic theory and mathematical statistics. Therefore courses 31a and 32b should be required of anyone in electing mathematical theory as his special field.

(4) No further steps need be taken at present. It seems best to see what the response will be before attempting to organize a special graduate course. Any individual cases calling for special attention can be dealt with, under the proposed regulation, as our courses now stand.

Source:  Harvard University Archives. Department of Economics, Correspondence and papers 1930-1961. (UAV349.11), Box 13.

_____________________

Statistics Courses offered in the Department of Economics
at Harvard, 1934-35

Economics 31a 1hf (formerly Economics 41a). Theory of Economic Statistics, I

Half-course (first half-year). Mon., Wed., Fri., at 9. Professor Crum and Asst. Professor Frickey.
Economics 1a, or its equivalent, is a prerequisite for this course.

Economics 31b 2hf (formerly Economics 41b). Theory of Economic Statistics, II

Half-course (second half-year). Mon., Wed., Fri., at 9. Professor Crum and Asst. Professor Frickey.
Economics 1a, or its equivalent, is a prerequisite for this course.

Economics 32b 2hf (formerly Economics 42). Foundations of Statistical Theory

Half-course (second half-year). Tu., Th., 3 to 4.30. Professor E. B. Wilson.
Economics 31and one year of Calculus are prerequisites for this course.

Source: Announcement of the Courses of Instruction offered by the Faculty of Arts and Sciences, 1933-34(second edition), Official Register of Harvard University, Vol. XXX, No. 39 (September 20, 1933), p. 128.

_____________________

Undergraduate Mathematics Courses
at Harvard, 1934-35

Mostly Freshmen

[Mathematics] A. Professors J. L Coolidge et al. — Analytic Geometry; Introduction to the Calculus.

Mostly Sophomores

[Mathematics] 2. Professors Graustein et al. — Differential and Integral Calculus; Analytic Geometry.

Mostly Juniors

[Mathematics] 5a1hf. Professor Morse. — Differential and Integral Calculus (advanced course), Part I

[Mathematics] 5a2hf. Professor Morse. — Differential and Integral Calculus (advanced course), Part II

 

Source: Harvard University. Report of the President of Harvard College, 1934-35, p. 86.

 

Images:  Left to right: William Leonard Crum, Joseph A. Schumpeter, Edwin Bidwell Wilson. From the 1934 (Crum) and 1939 (Schumpeter and Wilson) Harvard Class Albums.

 

 

Categories
Chicago Exam Questions Fields

Chicago. Industrial Organization Prelim. 1977

 

The following five questions come from what appears to be a draft of the prelim exam in industrial organization for the Spring of 1977 that is found in the George Stigler papers at the University of Chicago. The draft clearly has the title “Industrial Organization Prelim” but the date is a handwritten addition. Also there is no explicit “University of Chicago”  to be found, though given the location in George Stigler’s papers, this identification seems rather certain.

___________________

[handwritten note:  5-2-77]

Industrial Organization Prelim

Answer all questions:

  1. It is sometimes alleged that periods of economic depression are more conducive to the growth of economic regulation than prosperity. Develop a theory which elaborates the link between the level of economic activity and the propensity to regulate. Include a discussion of whether the goals of regulatory agencies (old as well as new) are likely to differ with the level of economic activity.
  2. Sales of some firms are occurring at prices below average variable cost. Suppose there were no legal restrictions on merger. Under what conditions, if any, would the firms in the industry prefer merger as a means of reducing industry output?
  3. A recent treatise on antitrust law lists the following as among the factors favorable to collusion in an industry.
    1. No fringe of small buyers.
    2. Inelastic demand at competitive price.
    3. Entry takes a long time.
    4. Many customers.
    5. A standardized product.
    6. High ratio of fixed to variable costs.

Discuss for each factor the effect, if any, it has on probability of collusion.

  1. What problems for profit maximizing collusion among the firms in the book publishing industry would arise because of each of the following conditions:
    1. There are two classes of books, fiction and nonfiction. Publishers generally publish both types although some publishers specialize in nonfiction.
    2. Sales and profits from n fictional books behave like n independent random variables drawn from the same distribution. Sales of a given fictional book in a given year are independent of the sales in the previous year. There is a positive probability of sales coming to a halt in a given year and remaining zero thereafter.
    3. Nonfiction is of two types, textbooks and “how-to-do-it” books. The expected number of years of positive sales for a nonfiction book is greater than for a fiction book.
    4. Retail outlets and mail order sales are the only channels of distribution to the final users of books.
    5. The cost function of a book publisher is proportional to the number of titles and the quantity printed of each title.
    6. Every title has the protection of a copyright. Assume that the Xerox machine and similar devices do not exist.
    7. Anyone can arrange to have a book printed by a printing company and can arrange for its distribution.
  2. How do you explain the following empirical findings for manufacturing industries?

Let

Nit= number of companies in the 4-digit industry i in year t
Cit= 4-firm concentration ratio, industry i, year t.
Qit= index of real output industry i, year t
Rit= measured rate of return of all firms in industry i, year t.

    1. For each t, holing log Nitconstant, Ritis an increasing function of Cit.
    2. For each t, holding Citconstant, Ritis an increasing function of log Nit.
    3. For each t, Cit and log Nitare negatively correlated.
    4. Between 1947 and 1967 the correlation between the change in Citand the change in log Nitis 0.55.
    5. Between 1947 and 1967 the correlation between the change in Citand the change in log Nitis zero. The correlation is also zero between the change in log Qitand the change in Cit.
    6. There has recently been renewed interest in the social optimality of various devices for the public regulation of pollution. Among popular proposals to deal with the problem: emission taxes, subsidies for pollution control, transferable rights to emit pollutants, maximum limits on pollution discharges from each source. Assume that the optimality conditions for public regulation have been met. Evaluate the relative efficiency of these four devices and any others you wish to add to the list.

Source: University of Chicago Archives. George Stigler Papers, Addenda. Box 33, Folder “Exams & Prelim Questions”.

Image Source:  George Stigler page at the University of Chicago Booth School of Business website.

Categories
Fields Harvard Suggested Reading Syllabus

Harvard. Consolidated undergraduate and graduate public finance syllabus. Butters and Soloway, 1954-55

 

Providing a ten page transcription of a course syllabus is a daunting task. It does have the useful side-effect of forcing me to read the syllabus closely and I still labor under the hope that something of potential future significance will lodge itself somewhere in my subconscious, ready to go if ever summoned. Of course having a digitized transcript allows us to easily search the growing sample of course syllabi already transcribed at Economics in the Rear-view Mirror. 

Harvard economics Ph.D.’s on the economics department faculty in the mid-1950’s, J. Keith Butters and Arnold M. Soloway, are listed on the public finance syllabus below that was distributed as a consolidated reading list for the undergraduate and graduate versions of the course taught in 1954-1955. I am not sure what to make of the fact that only Butters’ name appears in the enrollment report included with the annual report of the President of Harvard College.

P.S. The mid-year (January) and end-year (May) final exams have been transcribed and posted in a later post.

_______________________

Course Enrollments

[Economics] 151. Public Finance. Associate Professor Butters. Full course.

(W) Total 30: 15 Seniors, 9 Juniors, 1 Sophomore, 4 Other Graduates, 1 Other
(S) Total 27: 14 Seniors, 11 Juniors, 1 Sophomore, 1 Other Graduate

[Economics] 251 Public Finance. Associate Professor Butters. Full course.

(F) Total 19: 7 Graduates, 8 Other Graduates, 1 Radcliffe, 3 Special
(S) Total 16: 6 Graduates, 7 Other Graduates, 1 Radcliffe, 2 Special

Source: Harvard University. Report of the President of Harvard College 1954-1955, pp. 90, 93.

_______________________

Economics 151 and 251
PUBLIC FINANCE
Fall Term, 1954-1955

Professors Butters and Soloway

NOTE: Readings under the heading “Required” are required for Economics 151. Students in Economics 251 are required to read the asterisked assignments and to be generally familiar with the substance of the material covered in the other required assignments for Economics 151.

The following general studies and texts are suggested for reference throughout the course. Specific assignments on various topics are made from some of these sources.

General Texts and Treatises on Public Finance:

Blough, Roy, The Federal Taxing Process

Brownlee, O. H. and Allen, E. D., Economics of Public Finance, (Second Edition)

Due, John F., Government Finance

Groves, H. M., Financing Government (Third Edition) [Fifth edition]

Groves, H. M., Viewpoints on Public Finance

Hicks, U. K., Public Finance

Pigou, A. C., A Study in Public Finance

Poole, K. E., (Editor), Fiscal Policies and the American Economy

Schultz, W. J. and Harriss, C. L., American Public Finance [Third edition, before Harriss]

Somers, H. M., Public Finance and National Income

 

Serial Publications and Periodicals:

Annual Reports of the Secretary of the Treasury

Budget Messages of the President

Economic Reports of the President and Economic Reviews of the Council of Economic Advisers

Proceedings of the National Tax Association

National Tax Journal

Taxes, The Tax Magazine (Published by Commerce Clearing House, Inc.)

The loose-leaf tax services published by Commerce Clearing House, Inc. and Prentice-Hall, available in the Law Library

 

September 28: Nature and Scope of Government Finance

Required

*Brownlee and Allen, Economics of Public Finance, Second Edition, pp. 3-22

*Colm, Gerhard, “Why Public Finance,” National Tax Journal, Sept. 1948, pp. 193-206

*Due, Government Finance, Ch. 1, pp. 1-16

Suggested

*Hicks, Public Finance, Ch. 1, pp. 1-16

Groves, Financing Government, Ch. 1, pp. 1-8

 

September 30 – October 2: Concepts of Justice

Required

*Due, Government Finance, Ch. 7, pp. 114-133

*Simons, Henry, Personal Income Taxation, Ch. 1, pp. 1-40

*Blough, The Federal Taxing Process, Ch. 15, pp. 382-408

Suggested

Pigou, A. C., “Some Aspects of Welfare Economics,” American Economic Review, June 1951, pp. 287-302

*Pigou, A Study in Public Finance, Part II, Chs. 1-7, pp. 40-93

*Robbins, L., “Interpersonal Comparisons of Utility,” Economic Journal, December 1938, pp. 635-641

*Wright, D. Mc., “Income Redistribution Reconsidered,” Income, Employment and Public Policy, edited by Metzler, L. Pp. 159-176

Blum, W. J., and Kalven, Harry, The Uneasy Case for Progressive Taxation

Shehab, F., Progressive Taxation: A Study in the Development of the Progressive Principal in the British Income Tax

 

October 5 – October 16: The Budget

Required

Groves, Financing Government (Third Edition), pp. 509-527

Schultz and Harriss, American Public Finance, pp. 131-151

*Smithies, Arthur, The Determination and Control of Federal Expenditures (mimeographed volume), Chs. I-VI (128 pages)

*Smith, Harold D., The Management of Your Government, Chs. 5-7, pp. 54-102

*March, Michael, “A Comment on Budgetary Improvement in the National Government,” National Tax Journal, June 1952, pp. 155-173. Also, “Reply to Mr. March” by Herman Loeffler, same issue, pp. 174-175

*The Budget of the United States Government for the Fiscal Year Ending June 30, 1955, pp. M5-M104 and A3-A16. (This assignment can be scanned rather than studied carefully as to matters of detail.)

*National Income, 1951 (A Supplement to the Survey of Current Business) pp. 10-18, 21-34, 42-43, 46-49

*Tax and Expenditure Policy for 1950, Committee for Economic Development, pp. 35-41

Suggested

Hicks, J. R., The Problem of Budget Reform

Hansen, A. H., Fiscal Policy and Business Cycles, Ch. 10, pp. 186-222

Key, V. O., “The Lack of a Budgetary Theory,” American Political Science Review, Volume 34 (December 1940), pp. 1137-1144

U.S. Commission on Organization of the Executive Branch of the Government, Budget and Accounting, Parts I and II, pp. 7-31, 77-84

U.S. Commission on Organization of the Executive Branch of the Government, Task Force Report on Fiscal, Budgeting, and Accounting Activities (Appendix F), pp. 37-38

Loeffler, Herman C., “Alice in Budget-Land,” National Tax Journal, March 1951, pp. 54-64

Fieldler, Clinton, “Reform of the Legislative Budget,” National Tax Journal, March 1951, pp. 65-76

Burkhead, Jesse, “The Outlook for Federal Budget-Making,” National Tax Journal, December 1949, pp. 289-299

*Smithies, A., The Determination and Control of Expenditures, Chs. VII-XII and Ch. XVIII (Mimeographed)

Dirks, F. C., “Recent Progress in the Federal Budget,” National Tax Journal, June 1954, pp. 141-154

 

October 19 – November 6: Expenditures

Required

*Due, Government Finance, Chs. 2-6, pp. 17-113

*Musgrave, R. A. and Culbertson, J. M., “The Growth of Public Expenditures in the United States, 1890-1948,” National Tax Journal, June 1953, pp. 97-115

*”State and Local Government Receipt and Expenditure Programs,” Survey of Current Business, January 1953, pp. 11-16

*Douglas, P. H., Economy in the National Government, Chs. I-VIII, pp. 3-204

*Buchanan, J. S., “The Pricing of Highway Services,” National Tax Journal, June 1952, pp. 97-106

Studenski, “Federal Grants-in-Aid,” National Tax Journal, September 1949, pp. 193-214

*Newcomer, Mabel, “State and Local Financing in Relation to Economic Fluctuations,” National Tax Journal, June 1954, pp. 97-109

*Maxwell, J. A., “The Equalizing Effects of Federal Grants,” Journal of Finance, May 1954, pp. 209-215

*Stark, John R., “Equities in the Financing of Federal Old and Survivors Insurance,” National Tax Journal, September 1953, pp. 286-292

Suggested

*Maxwell, J. A., Federal Grants and the Business Cycle, Chs. I-IV, pp. 1-99

*Clark, C., “Public Finance and Changes in the Value of Money,” Economic Journal, December 1945, pp. 371-389

*Pechman, J. A., and Mayer, Thomas, “Mr. Colin Clark on the Limits of Taxation,” Review of Economics and Statistics, August 1952, pp. 232-242; and Smith, D. T., “Note on Inflationary Consequences of High Taxation,” Ibid., Pp. 243, 247

*Goode, Richard, “And Economic Limit on Taxes: Some Recent Discussions,” National Tax Journal, September 1952, pp. 227-233

*Pigou, A. C., A Study in Public Finance, Chs. I-V, pp. 1-34

Machlup, F., “The Division of Labor between Government and Private Enterprise,” American Economic Review, 1943 Supplement, pp. 87-104

Hansen, A. H., and Perloff, H. S., State and Local Finance in the National Economy, Chs. 2 and 8

Hicks, J. R. and Hart, A. G., The Social Framework of the American Economy, Ch. XIII, pp. 174-185

Bowen, H. R., Toward Social Economy, Ch. 18

Backman, Jules and Kurnov, Ernest, “Pricing of Government Services,” National Tax Journal, June 1954, pp. 121-140

 

November 9 – November 30: Fiscal Policy

Required

*Smithies, Arthur, “Federal Budgeting and Physical Policy,” in A Survey of Contemporary Economics (edited by Howard S. Ellis), Ch. 5, pp. 174-209

Hansen, A. H., Business Cycles and National Income, Ch. 12, pp. 195-207

(Note: Read one or two of the following four sources)

(1) Gordon, R. A., Business Fluctuations, Ch. 18, pp. 525-544

(2) Brownlee, O. H. and Allen, E. D., Economics of Public Finance, 2nd edition, Chs. VI-VIII, pp. 94-140

(3) Musgrave, R. A., “Fiscal Policy, Stability, and Full Employment,” Public Finance and Full Employment (Postwar Economic Studies No. 3, Board of Governors of Federal Reserve System), pp. 1-21

(4) Due, Government Finance, Chs. 25-26, and 28, pp. 470-505, and 524-550

*Hart, A. G., Money, Debt and Economic Activity, Second Edition, Chs. XXVII, XXVIII, and XXIX, pp. 448-495

*Hicks, U. K., Public Finance, Ch. XVII, pp. 316-336

*Committee for Economic Development, Taxes and the Budget: A Program For Prosperity in a Free Economy (November 1947), especially pp. 9-34

*Blough, Roy, “Political and Administrative Requisites for Achieving Economic Stability,” American Economic Review, May 1950, pp. 165-177

*Lerner, A. P., The Economics of Control, Ch. 24, pp. 302-322

*Pechman, Joseph A., “Yield of the Individual Income Tax During a Recession,” National Tax Journal, March 1954, pp. 1-16

Suggested

*Wallich, H. C., “Income Generating Effects of a Balanced Budget,” Quarterly Journal of Economics, November 1944, pp. 78-91

*Musgrave, R. A., and Painter, M. S., “The Impact of Alternative Tax Structures on Personal Consumption and Saving,” Quarterly Journal of Economics, August 1948, pp. 475-499

*Margolis, Julius, “Public Works and Economic Stability,Journal of Political Economy, August 1949, pp. 277-292

Beveridge, W. H., Full Employment in a Free Society

Hansen, A. H., Fiscal Policy and Business Cycles

Terborgh, George, The Bogie of Economic Maturity

Hansen, A. H., “Some Notes on Terborgh’s ‘The Bogie of Economic Maturity,’” Review of Economics and Statistics, February 1946, and Terborgh’s reply R. E. S., August 1946

*”The Problem of Economic Instability,” A committee report, American Economic Review, September 1950, pp. 505-538 (sections pertaining to fiscal policy)

Bach, G. L., “Monetary-Fiscal Policy, Debt Policy, and the Price Level,” American Economic Review, May 1947, pp. 228-242

Bronfenbrenner, M., “Postwar Political Economy: The President’s Reports,” Journal of Political Economy, October 1948, pp. 373-391

*Clark, J. M., “An Appraisal of the Workability of Compensatory Devices,” American Economic Review, Proceedings, March 1939, reprinted in Readings in Business Cycle Theory, pp. 291-310

“Problems of Timing and Administering Fiscal Policy in Prosperity and Depression,” papers by E. E. Hagen and A. G. Hart; discussion by J. K. Galbraith, B. H. Higgins, W. S. Soytinski, and O. H. Brownlee, American Economic Review, May 1948, pp. 417-451

*Musgrave, R. A. and Miller, M. H., “Built-in Flexibility,” American Economic Review, March 1948, pp. 122-128

Musgrave, R. A., “Alternative Budget Policies for Pole Full Employment,” American Economic Review, June 1945, pp. 387-400

Clark, J. M., Economics of Planning Public Works

Lubell, “Efforts of Redistribution of Income on Consumers’ Expenditures,” American Economic Review, March 1947, pp. 157-170; Correction, December 1947, p. 930; Comment by J. M. Clark, p. 931

Burkhead, Jesse, “The Balanced Budget,” Quarterly Journal of Economics, May 1954, Pp. 191-216

 

December 2 – December 18: Government Debt and Debt Management

Required

Due, Government Finance, Chs. 24 and 27, pp. 445-469 and 506-523

Schultz and Harriss, American Public Finance, Chs. XXV-XXVII, pp. 615-704

*Lerner, A. P., “The Burden of the National Debt” in Income, Employment and Public Policy (Metzler, L., et al.), Pp. 255-275

*”How to Manage the Debt,” Symposium in Review of Economics and Statistics, February 1949, pp. 15-32

*Murphy, H. C., The National Debt in War and Transition, Chs. 18-19, pp. 249-288

*Thomas, Woodlief, “Lessons of War Finance,” American Economic Review, September 1951, pp. 618-631

*Abbott, C. C., The Federal Debt (Twentieth Century Fund, 1952), Ch. 6, pp. 89-112

Suggested

Abbott, op. cit., pp. 1-196

*Roosa, R. V., “Interest Rates in the Central Bank,” in Money, Trade and Economic Growth (In Honor of John Henry Williams), pp. 270-295

*Simons, H. C., “On Debt Policy,” Journal of Political Economy, December 1944, pp. 356-361, and “Debt Policy and Banking Policy,” Review of Economics and Statistics, May 1946, pp. 85-89; both reprinted in Economic Policy for a Free Society, pp. 220-239

*Musgrave, R. A., “Credit Controls, Interest Rates and Management of Public Debt,” in Income, Employment and Public Policy (Metzler, L., At all.), Pp. 221-254

Harris, S. E., The National Debt and the New Economics

Committee on Debt Policy, Our National Debt

Seltzer, L. H., “Is a Rise in Interest Rates Desirable or Inevitable?” American Economic Review, December 1945, pp. 831-850

Roosa, R. V., “Integrating Debt Management and Open Market Operations,” American Economic Review, Supplement, May 1952, pp. 214-235

Wallich, H. C., “Debt Management as an Instrument of Economic Policy,” American Economic Review, June 1946, pp. 292-310

Bach, G. L., “Monetary-Fiscal Policy Reconsidered,” Journal of Political Economy, October 1949, pp. 383-394

Tobin, James, “Monetary Policy and the Management of the Public Debt: The Patman Inquiry,” Review of Economics and Statistics, May 1953, pp. 118-127

Burgess, W. Randolph, “Federal Reserve and Treasury Relations,” Journal of Finance, March 1954, pp. 1-11

 

*  *  *  *  *  *  *  *  *  *  *

Economics 151 and 251
PUBLIC FINANCE
Spring Term, 1954-1955

Professors Butters and Soloway

Note: Readings under the heading “Required” are required for Economics 151. Students in Economics 251 are required to read the asterisked assignments and to be generally familiar with the substance of the material covered in the other required assignments for Economics 151. References in Shultz and Harriss, American Public Finance, refer to the new 6thedition.

 

February 3-10: General Introduction to Taxation in the United States.

Required:

Shultz, W. J., and Harriss, C. L., American Public Finance, Chapters 7, 9, 10, 11.

Groves, Harold, Viewpoints on Public Finance, Chapter 1.

Lerner, A. P., Economics of Control, Chapter 24 (review).

Suggested:

*Bullock, C. J., Readings in Public Finance, Chapters VIII-IX.

Paul, Randolph E., Taxation in the United States (1954).

Ratner, Sydney, American Taxation, Its History as a Social Force in Democracy (1942).

Dewey, Davis R., Financial History of the United States.

 

February 10-17: Personal Income Taxation.

Required:

Shultz, W. J., and Harriss, C. L., American Public Finance, Chapters 12, 13.

*Simons, H. C., Personal Income Taxation, Chapter I (reread), Chapters II, III (passim), IV-VI, VIII, X.

Groves, H. M., Financing Government, 3rdedition, Chapter 9.

Your Federal Income Tax, Bureau of Internal Revenue.

Suggested:

*National Tax Journal, March 1955, articles by Professor Shoup, Brown, and Pechman.

*Vickrey, W. S., Agenda for Progressive Taxation, Chapters 1, 2, 4, 6 (passim), 12, 13, 14.

Fisher, I., and Fisher, H. W., Constructive Income Taxation, A Proposal for Reform, Chapters 1, 5, 7, 8, 9, and 21.

Holt, C. G., “Averaging of Income for Tax Purposes: Equity and Fiscal-Policy Considerations,” National Tax Journal, December 1949.

*Musgrave, R. A., and Tun, Thin, “Income Tax Progression, 1929-48”, Journal of Political Economy, December 1948, pp. 498-514.

Farioletti, Marius, “The 1948 Audit Control Program for Federal Income Tax Returns”, National Tax Journal, June 1949, pp. 142-150.

Farioletti, Marius, “Some Results from the First Year’s Audit Control Program of the Bureau of Internal Revenue”, National Tax Journal, March 1952, pp. 65-78.

Blakey, R. G., and Blakely, G. C., The Federal Income Tax.

Magill, Roswell, Taxable Income.

Prentice-Hall, Federal Tax Course – 1954, Chapters 1-3.

 

February 19-24: Capital Gains Taxation.

Required:

*Seltzer, L. H., The Nature and Tax Treatment of Capital Gains and Losses, Chapters 1, 2, 4, 9, 11.

Groves, H. M., Financing Government, 3rd edition, pp. 172-177.

*Simons, H. C., Personal Income Taxation, Chapter VII.

Suggested:

*Vickrey, W. S., Agenda for Progressive Taxation, Chapter 5.

Capital Gains Taxation (A Tax Institute Symposium) (passim).

Federal Income Tax Treatment of Capital Gains and Losses (A Treasury Tax Study), 1951.

Groves, H. M., Viewpoints on Public Finance, pp. 151-158.

Prentice-Hall, Federal Tax Course – 1954, Chapters 4-6.

 

February 26-March 5: Corporation Income Tax.

Required:

Shultz, W. J., and Harriss, C. L., American Public Finance, pp. 311-320.

*Goode, Richard, The Corporation Income Tax, Chapters 1-9, 11, 18.

*Thompson, L. E., and Butters, J. K., “Effects of Taxation on the Investment Policies and Capacities of Individuals”, Journal of Finance, May 1953, Pp. 137-151.

*Smith, D. T., “Taxation and Executives”, Proceedings of the National Tax Association, 1951, pp. 232-250.

*Brown, E. C., “Business-Income Taxation and Investment Incentives”, Income, Employment, and Public Policy (Essays in Honor of Alvin H. Hansen), pp. 300-316.

Butters, J. K., and Lintner, J., Effect of Federal Taxes on Growing Enterprises, Chapters I-VII, VII and IX passim.

Suggested:

Prentice-Hall, Federal Tax Course – 1954, Chapters 21-23.

Smith, D. T., and Butters, J. K., Taxable and Business Income, Forward, Introduction, and Chapter 1.

*Slitor, Richard E., “The Corporate Income Tax: A Re-evaluation”, National Tax Journal, December 1952, pp. 289-309.

*Domar, E. D., and Musgrave, R. A., “Proportional Income Taxation and Risk-Taking”, Quarterly Journal of Economics, May 1944, pp. 388-422.

Butters, J. K., Effects of Taxation on Inventory Accounting and Policies, Chapters I, IV, V.

Butters, J. K., Thompson, L. E., and Bollinger, L. L., Effects of Taxation on Investments by Individuals, Chapters I-VI.

Smith, D. T., Effects of Taxation on Corporate Financial Policy, Chapters I, VI-IX.

*Smith, D. T., “Corporate Taxation and Common Stock Financing”, National Tax Journal, September 1953, pp. 209-225.

Brown, E. See., Effects of Taxation on Depreciation Adjustments for Price Changes, Chapters I-IV.

*Eldridge, D. H., “Tax Incentives for Mineral Enterprise”, Journal of Political Economy, June 1950, pp. 222-240.

Economic Effects of Section 102 (Tax Institute Symposium, 1951).

 

March 8-10: Integration of Personal and Corporate Income Taxation.

Required:

*Goode, Richard, The Corporation Income Tax, Chapter X.

*Simons, H. C., Personal Income Taxation, Chapter IX.

Suggested:

*The Postwar Corporation Tax Structure, U.S. Treasury Study.

How Should Corporations be Taxed?, A Tax Institute Symposium.

“Final Report of the Committee on the Federal Corporation Income Tax”, Proceedings of the National Tax Association, 1950, pp. 54-76.

Lent, G. E., The Impact of the Undistributed Profits Tax, 1936-1937.

 

March 12-15: Excess Profits Taxation.

Required:

*Hart, A. G., and Brown, E. C., Financing Defense, Chapter 7.

*Blough, Roy, “Measurement Problems of the Excess Profits Tax”, National Tax Journal, December 1948, pp. 353-365.

*”Symposium on the Excess Profits Tax”, National Tax Journal, September 1951, pp. 219-36.

Tax Institute, Excess Profits Tax, Parts 1 and 3, and pp. 119-141.

Suggested:

Oakes, E. E., “Excess Profits Tax Amendments”, National Tax Journal, March 1952, pp. 53-64.

Hicks, J. R., Hicks, U. K., and Rostas, L., The Taxation of War Wealth, Chapters 1, 4-7.

 

March 14-19: Estate and Gift Taxation.

Required:

Schultz, W. J., and Harriss, C. L., American Public Finance, Chapter 20.

*Groves, H. M., Viewpoints on Public Finance, Nos. 44, 46, 47, and 48 (all in Chapter 5).

*Butters, J. K., Lintner, J., and Cary, W. L., Effects of Taxation on Corporate Mergers, Chapters I-III and V.

Bloch, Henry S., “Economic Objectives of Gratuitous Transfer Taxation”, National Tax Journal, June 1951, pp. 139-147.

Suggested:

*Surrey, Stanley S., et al., “A Critique of Federal Estate and Gift Taxation”, California Law Review, March 1950. (Introduction by Stanley Surrey, Pp. 1-27, required for graduate students; remainder optional.)

*Federal Estate and Gift Taxes– A Proposal for Integration and for Correlation with the Income Tax. (A joint study by an advisory committee to the Treasury Department and the Office of the Tax Legislative Council, 1947) (Sections I and II and remainder, passim. Required for graduate students).

Keith, E. Gordon, “How Should Wealth Transfers Be Taxed?”, American Economic Review, May 1950, pp. 379-390.

Wedgewood, Josiah, The Economics of Inheritance, especially Chapters 9-11.

 

March 22-31: Taxes on Consumption.

Required:

Schultz, W. J., and Harriss, C. L., American Public Finance, Chapters 8, 16.

*Groves, H. M., Viewpoints on Public Finance, Nos. 58, 59, 60, 64.

Soloway, A. M., “The Purchase Tax and Fiscal Policy”, National Tax Journal, December 1951.

Suggested:

Due, John F., “American and Canadian Experience with the Sales Tax”, The Journal of Finance, September 1952.

*Due, John F., “Toward A General Theory of Sales Tax Incidents”, The Quarterly Journal of Economics, May 1953.

Pao Lun Cheng, “A Note on the Progressive Consumption Tax”, The Journal of Finance, September 1953.

Soloway, Arnold M., “Economic Aspects of the British Purchase Tax”, Journal of Finance, May 1954.

*Hicks, U. K., Public Finance, Chapters IX and X.

Hart and Brown, Financing Defense, Chapter 4.

 

April 12-23: Intergovernmental Tax Problems.

Required:

Shultz, W. J., and Harriss, C. L., American Public Finance, Chapters 23, 24, 18, 19.

*Groves, H. M., Postwar Taxation and Economic Progress, Chapter 12.

*State-Local Relations, The Council of State Governments, Report of the Committee on State-Local Relations, 1946, Parts 3 and 4; Parts 1, 2, 5, and 6 passim.

*Federal State Local Tax Correlation; Symposium of the Tax Institute, 1953. Chapters I, II, III, VII, VIII, XVIII.

Suggested:

Groves, H. M., Postwar Taxation and Economic Progress, Chapter 12.

Groves, H. M., Viewpoints on Public Finance, Chapter 2.

*George, Henry, Progress and Poverty.

Hansen and Perloff, State and Local Finance in the National Economy.

*National Tax Journal, December 1951, pp. 341-371.

 

April 26-May 5: Burden of Taxation.

Required:

*Musgrave, R. A., et al., “Distribution of Tax Payments by Income Groups”, National Tax Journal, March 1951, pp. 1-53.

*Tucker, Rufus S., “Distribution of Tax Burdens in 1948”, National Tax Journal, September 1951, pp. 269-283.

*Allen, E. D., and Brownlee, O. H., Economics of Public Finance, Chapter X.

*Tucker, R. S., “Distribution of Government Burdens and Benefits”, American Economic Review, May 1953, pp. 519-534.

Suggested:

*”Further Consideration of the Distribution of the Tax Burden”, National Tax Journal, March 1952, pp. 1-39.

Poole, K. E., Fiscal Policies and the American Economy (Chapter VIII, “The Fiscal System, The Distribution of Income, and Public Welfare” by John H. Adler), pp. 359-409.

 

Source: Harvard University Archives. Syllabi, course outlines and reading lists in Economics, 1895-2003. Box 6, Folder “Economics, 1954-1955”.

Image Source: J. Keith Butters from Webpage of the Harvard Business School Baker Library Historical Collection “Edwin H. Land & the Polaroid Corporation: The Formative Years”.

Categories
Exam Questions Fields Harvard Undergraduate

Harvard. Three Undergraduate Economic Field Exams, 1942

 

The Harvard undergraduate economics departmental exam and the essay topics for 1942 were transcribed for the previous post. Below we have three field exams for money & finance, market organization & control, and labor economics & social reform from the same year. In the Randall Hinshaw papers at Duke I did not find field exams for statistics & accounting or economic history that I suspect would have also been offered (judging from Part II of the economics departmental exam).

________________________

DIVISION OF HISTORY, GOVERNMENT, AND ECONOMICS
Department of Economics
May 6, 1942

DIVISION SPECIAL EXAMINATION
Money and Finance
(Three hours)

PART I
(About one hour)

  1. Write an essay on ONE of the following topics:
    1. monetary conditions of full employment equilibrium,
    2. the functions and importance of the Federal Reserve System in the 1920’s, the 1930’s, and today,
    3. investment banking by commercial banks – theory and practice in the past and future,
    4. international monetary problems after the last war, and after this war,
    5. modern improvements on the classical theory of international trade,
    6. ideas for post-war liberation and control of international trade – conditions of progress in respect of justice to all nations and prosperity for all,
    7. modern federal taxation in peace and war times – functions, and types of taxes and tax programs required,
    8. ways of mitigating the undesirable future consequences of our mounting national debt,
    9. effects of the war on financial problems of state governments,
    10. the background of the modern vogue of monetary management and deficit finance, in fundamental economic changes over recent decades,
    11. prospective war-time and immediate post-war changes in America, in demand and supply conditions for investment funds and real capital,
    12. post-war problems and prospects in Anglo-American economic relations.

 

PART II
(About one hour)

All students must answer TWO questions. If you are a candidate for honors, at least ONE of these two must be a starred question.

  1. (*) “The spectre of ‘secular stagnation’, which threatened the capitalist world of the 1930’s, is being exorcised by this war and will probably not return after it, at least for some decades.”
  2. (*) Outline succinctly, and explain and discuss as fully as your time allows, what you regard as the best analysis – either one writer’s or your own compilation – of the fundamental causes of the business cycle.
  3. Explain, and discuss critically, several different concepts of “velocity” and “hoarding” found in the modern literature of monetary theory.
  4. “Just as banking policy was unable, in the 1930’s, to play any important part in producing recovery, it is now unable, for opposite and parallel reasons, to play any important part in combating war inflation.”
  5. (*) Discuss the economic and other causes of the world-wide growth of new nationalistic restrictions on international trade, in the interval between the last war and the present war.
  6. (*) “As a stabilizer of the monetary basis of international trade, nothing short of one world currency under the management of a central, international authority, can be an effective substitute for the 19 century’s international gold standard.”
  7. Discuss the effects which the “lend-lease” arrangements through which this country is aiding its allies in the war, are likely to have on our foreign trade, economic relations with the outer world, and economic position in the post-war period.
  8. “If country A has strong labor unions which force up and hold up wage-costs in all its industries, while country B enjoys cheap labor together with industries as modern and well mechanized as those of A, progressive depreciation by A of the external value of its currency is its only means of maintaining competition with B in world markets.”
  9. (*) Discuss the relative merits of compulsory savings plans, a further lowering of exemptions from the personal income tax, and a general sales tax, as methods of diverting a larger share of war-time wages from consumption expenditure to investment in the war effort.
  10. (*) “Federal expenditures on welfare projects, or benefaction’s to the under-privileged, are a national luxury which must be sacrificed to the war effort.”
    “No; on the contrary, the war increases our obligation to all we can for the well-being of our poorest citizens; for in relation to the war effort, their morale is more important than are all economies, which would benefit only the over-privileged – whose patriotism, we hope, will stand the strain.”
  11. Discuss the merits of the view that in wartime the income tax should be supplemented by a special, progressive tax on all increases of individual incomes above the average levels of the same incomes in a group of pre-war years.
  12. “The chief danger in severe taxation of business profits in wartime is that of causing under maintenance of industrial plant, to the extent of making the country pay for the war to largely by consuming its capital.”

 

PART III
(About one hour)

(Answer TWO questions)

  1. “Future alternations of prosperity and depression are unlikely to occur with the nearly exact regularity or periodicity, which has made the term ‘business cycle’ appropriate in the past. The ‘cycle’ in that sense was one of the regularities peculiar to a quasi–automatic, laissez-faire capitalism.”
  2. “Money and finance are of no importance in modern war; only physical resources and production count. The Axis countries are already bankrupt, but it makes no difference. And we, in order to win the war, will have to give our physical production experts – not our monetary and fiscal experts – a free hand.”
  3. “America is sure to have, before the war ends, an inflation that will largely wipe out the real incomes and wealth of all its professional people and small savers – the backbone of the middle-class – and divide the spoils between rich speculators and skilled, industrial wage-earners. And that will make impossible the future maintenance of the country’s conservative-liberal, political tradition.”
  4. “The effort to knit the Latin American economies into ours, and make the Western Hemisphere a largely unified and self-sufficient, regional economy, cannot succeed in any large and lasting way. Our principal, natural economic ties are with Europe, and so are those of the Latin American countries; and these old, natural tendencies will reassert themselves after the war.”
  5. “By ending the imperialism of the white race in the Orient, the war is ending what have been essential factors in the prosperity of England, Holland, and America – exploitation of cheap Oriental labor and rich natural resources acquired at little cost, and a market for ‘dumping’ industrial surpluses, so as to make something near to full employment in the Western countries compatible there with excessive prices for the same industrial products.”
  6. “The spread of industrialism throughout the world does not merely alter the incidence everywhere of ‘comparative advantage’, and the international division of labor; it increases the diversity of productive powers and the self-sufficiency of every country, and thus radically diminishes the total importance of international trade.”
  7. “Financial, or monetary and fiscal manipulations cannot save capitalism. They could, if the right manipulators could work freely and not be defeated by a ‘strike’ on the part of Capital. But every attempt, in a time of depression, to redistribute money income and thus restore consumption and employment, always will be defeated by the further decline of investment due to the fears of the capitalists, who fear what immediately attacks their positions more than they fear the eventual, socialist revolution that is certain to result in time from an unrelieved, severe depression.”
  8. “In opposition to the nineteenth century orthodox explanation and defense of interest as a payment necessary to induce, through saving, enough creation of real capital, Keynes in effect revives the basic idea and resulting attitude of Aristotle and the medieval writers against ‘usury’. Like them, he sees in the demand for interest only the reluctance of the rich to part with their money hoards, and thus makes it the villain of the economic drama.”
  9. “In the economic world, the ‘real’ in contrast with the ‘monetary’ factors do indeed determine, as the older economists thought, what everyone must do in order to reach true equilibrium. Where they went wrong was in supposing that everyone always does fairly soon reach true equilibrium, that is, adjustment to realities; that deceptive, monetary changes have only very brief, transitional, or ‘short run’ consequences. Money is much more important than they thought it was, because the truth is that activities supported only by illusions, of monetary origin, prolong and aggravate those illusions and themselves in a cumulative fashion until unreality, or non-adjustment to reality, becomes so drastic that it collapses violently and then gives way, only, to a like, prolonged departure from reality in the opposite direction.”

________________________

DIVISION OF HISTORY, GOVERNMENT, AND ECONOMICS
Department of Economics
May 6, 1942

DIVISION SPECIAL EXAMINATION
Market Organization and Control
(Three hours)

PART I
(About one hour)

  1. Write an essay on ONE of the following topics:
    1. corporate profits,
    2. the problem of converting plants to war production,
    3. some recent developments in the study of costs of production,
    4. the war and American agriculture,
    5. why farmers are poor,
    6. the “parity” concept in agricultural policy,
    7. a wartime plan for the railroads,
    8. the future of private and public ownership in the public utility field,
    9. public utility rate-making: science or art?
    10. the relation of price control and rationing to fiscal policy,
    11. bureaucracy in industry and government,
    12. the Supreme Court and the regulation of economic life.

 

PART II
(About one hour)

All students must answer TWO questions. If you are a candidate for honors, at least ONE of these two must be a starred question.

  1. (*) Select any two American industries and compare their respective pricing methods and policies. Which seems to you more desirable from a public standpoint? Explain.
  2. Suppose you were put in charge of a trust fund with the duty of investing funds in corporate stock. What factors would you take into account in deciding which stocks to buy? Why?
  3. (*) Explain the relation, if any, between industrial price policies and the size of the national income.
  4. “The recent downward trend in the stock market is an utter absurdity from an economic point of view.” What facts and theories underlie this statement? Do you agree with it? Explain.
  5. (*) “We are now experiencing an agricultural revolution no less profound than the industrial revolution of 150 years ago.” Do you agree? Why or why not?
  6. (*) Discuss the chief problems of public policy connected with the growing and marketing of cotton.
  7. Discuss critically the recent agricultural policy of one foreign country.
  8. What are the principal changes that have been introduced in the methods and living conditions of American farmers by the internal combustion engine?
  9. (*) “Whenever you tried to define a public utility you will always come down finally to one and only one factor: discriminating monopoly.” What is a discriminating monopoly and what conditions favor its existence? Do you agree that discriminating monopoly is the distinguishing characteristic of public utilities? Explain.
  10. (*) What justification, if any, can be offered for the principle of railroad rate-making which attempts to equalize the competitive position of producers over a wide area?
  11. What conditions in the field of public utilities led to the passage of the Public Utility Holding Company Act of 1935?
  12. Discuss the relative merits of water-power and steam-power in the generation of electricity. Should it be public policy to favor one against the other (a) as a war measure, (b) in the post-war period? Explain.

 

PART III
(About one hour)

Answer TWO questions

  1. “It is an odd circumstance that capital fought for the right to incorporate, while labor fights against the compulsion to incorporate.” Discuss.
  2. “From an economic standpoint there is little to be said for excess profits taxation. As a method of controlling inflation it is obviously quite inadequate. Hence the only important consequence is an undermining of the financial position of precisely those corporations which are most essential in war production.” Discuss.
  3. Discuss the methods which have been employed in financing plant expansion requirements necessitated by the defense and war efforts. Why were these methods adopted? What is their significance for the post-war period?
  4. “The technical and managerial classes are slated to succeed the owners in the sequence of ruling classes.” Discuss.
  5. Some experts believe there is likely to be a great increase in the number and importance of corporate farms in the relatively near future. What are the reasons for this belief? Explain why you agree or disagree.
  6. Do you think direct control over wages is necessary to effective price control? Why or why not?
  7. Sketch the traditional policy of our government toward participation by American businessmen in international cartels and combines. Discuss the reasons for this policy and its results.
  8. “From the standpoint of economic organization, the Nazi economy represents the uninterrupted continuation of trends in German society which reach back at least to the 1870’s.” Discuss.

________________________

DIVISION OF HISTORY, GOVERNMENT, AND ECONOMICS
Department of Economics
May 6, 1942

DIVISION SPECIAL EXAMINATION
Labor Economics and Social Reform
(Three hours)

PART I
(About one hour)

  1. Write an essay on ONE of the following topics:
    1. wages and war inflation,
    2. the closed shop,
    3. should the 40-hour week be abolished during the war?
    4. the problem of migratory labor,
    5. an ideal system of unemployment insurance,
    6. a population policy for America,
    7. class struggle – reality or propaganda slogan?
    8. the probable effect of the war on American movements of social reform,
    9. can socialism be achieved by a gradual process of reform?
    10. labor and the anti-trust laws,
    11. trade unions and political action,
    12. labor in World War I.

 

PART II
(About one hour)

All students must answer TWO questions. If you are a candidate for honors, at least ONE of these two must be a starred question.

  1. (*) Discuss the benefits which one important C.I.O. union has won for its members, and the methods and policies by which it has won them.
  2. (*) Assume that a new industrial union enrolls all the workers in a particular industry, and succeeds in raising their wages. Make, and stayed clearly, your assumptions about all the main economic conditions (supply and demand conditions in the various markets) relevant to this problem; and on your assumptions, analyze the determination of the shares of the cost of paying for this wage-increase, which will be born in the end respectably by (1) the employers in the industry, (2) the consumers of the product, and (3) groups connected with other industries as workers, employers, or consumers.
  3. Discuss the history, methods, and achievements of union-management coöperation in one American industry where it has become established.
  4. What principles, as to policy and procedure, would you advise the federal war labor Board to adopt as its guiding principles in dealing with industrial disputes during the war period? Explain your reasons for each principal you propose.
  5. (*) Is the Malthusian theory of population wrong? If so, in what respects and why? If not, what is the evidence to support it?
  6. (*) Explain and evaluate the theory of non-competing groups.
  7. Can fascism (including Nazism) be called the “revolution of the middle class”? Explain.
  8. What, in your opinion, would be the chief economic effects of a cessation in population growth? Why?
  9. (*) Discuss critically Marx’s theory of capitalist crises.
  10. (*) What kind of a “new order” from an economic standpoint do the Nazis want to create?
  11. Discuss the main characteristics and results of economic planning in the Soviet Union.
  12. According to a number of economists, the price policy of a socialist society should be based on one single principle: equate price to marginal cost. Explain the meaning of this rule and argue for or against its general validity.

 

PART III
(About one hour)

Answer TWO questions

  1. Discuss the relative advantages and disadvantages, from the workingman’s standpoint, of the sales tax and a tax on wages deducted at the source as methods of closing the gap between outstanding purchasing power in the quantity of consumer goods available in the war economy.
  2. “Whether profit-sharing be but a slight modification of the ordinary capitalist system or contained within itself the germs of a true coöperative system need hardly be discussed in view of the fact that its history has been a record of repeated failure. The cause of failure in almost every case has been the apparent incompatibility of profit-sharing with trade unionism.” Discuss.
  3. What is to be said for stabilization of money wages as a goal of monetary policy?
  4. “Can even the most ardent free-trader doubt that in the post-war world American labor will continue to demand and deserve protection from cheap foreign labor?” Discuss.
  5. Discuss the economic problems of the construction industry, placing the kind of unionism which prevails there in its proper setting.
  6. Discuss the structure, problems and policies of the labor movement in backward or colonial countries.
  7. “There is no mistaking the economic foundations of race prejudice in the contemporary world.” Discuss.
  8. “Historically the connection between freedom of enterprise and freedom in other fields of thought and action is obvious. Must we not, then, assume that the destruction of free enterprise would likewise deprive us all our cherished liberties?” Discuss.

 

Source:  Duke University. David M. Rubenstein Rare Book & Manuscript Library. Economists’ Papers Archive. Papers of Randall Hinshaw. Box 1, Folder “Schoolwork, 1940s”.

Image Source: Harvard Square from the Tichnor Brothers Collection of postcards. Boston Public Library, Print Department.

Categories
Exam Questions Fields Harvard Undergraduate

Harvard. Undergraduate Departmental Examination and Essay Questions, 1942

 

 

The next post will provide transcriptions of three division special (i.e. field) examinations from 1942.

The 1939 departmental examination and  essay questions have been posted earlier.

______________________

DIVISION OF HISTORY, GOVERNMENT, AND ECONOMICS
Department of Economics
May 1, 1942

ESSAY PAPER
(One hour and a half)

Candidates for honors may write on ONE topic only. Others may, if they prefer, write on TWO topics. Please note on the front cover of the bluebook the number of each topic upon which you write.

  1. Economic imperialism.
  2. The pre-requisites of lasting peace.
  3. The economist who has most influenced your thinking.
  4. Some unsettled questions of economic science.
  5. Welfare economics.
  6. The relation of economics to sociology and political science.
  7. The distribution of wealth and income.
  8. The classical economists and their legacy.
  9. The nature and significance of general equilibrium analysis.
  10. Economic warfare.
  11. If Great Britain loses her empire.
  12. What killed laissez-faire?
  13. “The rise of political centralism is largely the product of economic centralism.”
  14. The relations and roles of the economic interests, and the social and cultural traditions, movements, and ideals, which are in conflict in the war.
  15. The American war effort and the profit system.
  16. Government controls which the American economy requires during the war, and those which it will require in the period of post-war adjustment.
  17. The applicability of traditional economic theory in explaining the course of economic life in totalitarian states.
  18. The future of capitalism.
  19. “The claim of economics to be a true science, like the modern physical sciences, must be given up as untenable.”
  20. Planned economies and human liberties.
  21. The value of training in economics, for success in business, and for good citizenship.
  22. “The physiologist’s task is not the physician’s; analysis and therapy are different; and economists, like physiologists, should confine themselves to explaining what happens, and leave the giving of advice to others.”

Source:  Duke University. David M. Rubenstein Rare Book & Manuscript Library. Economists’ Papers Archive. Papers of Randall Hinshaw. Box 1, Folder “Schoolwork, 1940s”.

___________________

DIVISION OF HISTORY, GOVERNMENT, AND ECONOMICS
Department of Economics
May 4, 1942

DEPARTMENTAL EXAMINATION
(Three hours)

Answer SIX questions; at least ONE question must be answered in each part, but not more than THREE questions may be taken in Part II. A senior may not take more than ONE question in that section of Part II which covers his special field.

PART I

  1. Define: elasticity of demand, unit elasticity, elastic demand, inelastic demand. Say weather, and explain why, you would expect the demand for each of the following commodities (in normal times) to be elastic or inelastic: automobiles, milk, tobacco, fur coats, window glass, oriental rugs, quinine, coal.
  2. Suppose that industries A, B, and C are all “purely competitive”, and that A has constant costs, B increasing costs, and C decreasing costs, for increasing outputs. If all three of these industries experience rapid, marked, and lasting increases of the public’s demands for their products, what will be (a) the immediate and (b) the ultimate effects upon the prices of the three different products? Explain your answers, and illustrate each case by the appropriate diagram. If now a cost-reducing invention (new method or machine) is generally adopted in each industry, show on your diagrams the effects of this on their cost conditions, outputs, and prices, and explain.
  3. Suppose a firm to be operating under these conditions:

Its total fixed cost is $1000 per day.
Its total operating cost for 1 unit output per day is $1000.00; for 2 units, $1800.00; for 3, $2550.00; 4, $3400.00; 5, $4500.00; 6, $6600.00. It can sell at price $1800.00, 1 unit; at $1500.00, 2 units; at $1250.00, 3 units; at $1100.00, 4 units; at $1000.00, 5 units; at $925.00, 6 units.

Infer from those figures, and draw on a diagram (as smooth curves) this firm’s average total unit cost, marginal cost, demand, and marginal revenue curves.
Now show on your diagram, and explain, the price and output required to maximize the firm’s profits.
Now assume “free entry” to the field, and that new competitors of this firm appear.
Show on your diagram, and explain, the ultimate effects of the new (increased) competition on this firm’s demand curve, output, average total unit cost, selling price, and profits.

  1. Explain as fully as you can, in terms of the relevant conditions of demand, supply, and marginal productivity, the present high wages of skilled workers in American war industries.
    To what extent, and how, do you think the efforts of trade unions make these wage-rates higher than they would be otherwise?
  2. In what principal ways do you think the war is affecting and likely to affect, while it lasts, the aggregate demand for and supply of capital and the level of interest rates within this country?
    What developments in the same respects do you think are most likely in the post—war period? Explain fully.
  3. Explain and discuss the significance of each of the following: total utility, law of diminishing utility, average and marginal utility, and consumers’ surplus.
  4. How would competition, if universally “pure”, tend to allocate resources, in a state of equilibrium of the whole economy?
    How is the equilibrium allocation altered by general prevalence of “monopolistic competition”?
    Explain concisely.
  5. Suppose that economic conditions in a country over a certain decade undergo the following changes. (1) The country’s population increases rapidly, while no additions are made to its territory or known natural resources. (2) Technological progress in all branches of production is steady and substantial; all innovations are capital-using, labor-saving inventions; physical outputs per man-hour of labor increase substantially. (3) A constant, rather high percentage of the national money income is annually saved and invested within the country. (4) Credit expansion is continually greater than the increase of total physical production, hence the price-level rises throughout the decade.
    Explain and discuss the probable, separate and joint effects of those developments on the absolute and relative shares of the national, real income respectively allotted, at the end as compared with the beginning of the decade, to (real) wages, economic rent, interest, and business profits. If you need to make assumptions more definite than those stated above, or additional assumptions, in order to reach definite conclusions, make clear the uncertainties in the problem as stated, and resolve them by explicit assumptions chosen as you please, at appropriate points in your discussion.

 

PART II
A
Statistics and Accounting

  1. Is it possible to devise an “ideal”, all-purpose, formula for price index numbers? Why or why not?
  2. What, in your judgment, are the greatest dangers that have to be guarded against in applying statistical methods to the available data of economic life?
  3. “Currently practiced accounting methods lead almost invariably to either overestimation or underestimation of true net earnings.” Explain carefully, indicating what is meant by “true net earnings” and why accepted accounting principles may lead to their misrepresentation. Do you think that in wartime, net earnings are likely to be overstated or understated?
  4. Answer concisely the following questions: (a) A corporation issues $100,000 par value stock to the promoters for nothing. In order to make the totals of the balance sheet equal, an item of “goodwill $100,000” is placed on the asset side. Assuming there is no reasonable ground for considering the “goodwill” to be actually valuable, how would you correct the balance sheet? (b) The amount of fixed assets – buildings and machinery – is less at the end of the year than at the beginning. What other changes would you expect to find on the balance sheet? Why? (c) In case a reappraisal of fixed assets shows a value in excess of value and it is desired to bring the appreciation into the books, how may this be done?

B
Modern Economic History

  1. What role would you assign to the National Banking System in the pattern of American business fluctuations from 1870 to 1914?
  2. Describe and explain the development of American tariff policy during the 19th century.
  3. Argue for or against the proposition that the Nazi economy is no more than the logical outcome of German economic policy from the time of Bismarck on.
  4. “The depression (1876-86) is, indeed, the watershed between the era of British industrial supremacy in the era of international competition.” Discuss.

 

C
Money and Finance

  1. Imagine that someone with no knowledge of economics asks you to explain to him, fully and clearly, why as an element of war finance government borrowing from the banks is peculiarly “inflationary”; and write out the explanation you would give.
  2. “Since government spending has become the main regulator of the volume and tempo of economic activity, Federal Reserve policy has become an academic subject of no real importance.”
  3. In a world at peace, with international trade proceeding normally, but with all countries on independent “paper standards” and exchanges “free” (with no fixed parities”, a position of general equilibrium and stable exchange rates has been reached. Now country A embarks, alone, on an internal monetary expansion which raises its price level.
    Trace and explain what effects, if any, this will tend to have on the balances of payments of A and other countries, foreign exchange rates, international transfers of products, factors, and “purchasing power”, and price levels in other countries. At what point, and how, will a new position of equilibrium be reached?
  4. “In the development of trade between an industrial nation, A, and an agricultural nation, B, both nations will gain by the trade, but the division of the gain will become unequal, in favor of A. The elastic demand for A’s products in B, and the inelastic demand for B’s products in A, will cause the terms of trade to shift in favor of A, as production in both countries in the trade between them expand.”
    Give a full and careful explanation of the concepts, assumptions, and reasoning suggested, and state any criticisms or qualifications that occur to you.
  5. Discuss the meaning and validity of the statement that a general sales tax is “regressive”; and the principal arguments for and against the view that this type of tax, even if undesirable in peace times, is peculiarly appropriate in wartime.
  6. “Our immense and upward-zooming federal debt is a prelude either to national bankruptcy, or else to socialism.”

 

D
Market Organization and Control

  1. Sketch the background, provisions, and chief consequences of the Sherman Anti-Trust Act.
  2. Is it possible for a Board of Directors to pursue a dividend policy which will consistently harmonize the interests of the corporation, its stockholders, and society as a whole? Explain.
  3. What are the methods which may be adopted to control war-time profits? What policy do you favor in this respect and why?
  4. “In the pricing of electrical energy no case can be made out on economic grounds for differential charges unless they are likely to lead to an improvement in the load factor, i.e., To a more uniform distribution of demand through time.” State your reasons for agreeing or disagreeing with this proposition.
  5. “Only a socialist has a right to complain about crop-restriction and price-raising in the field of agricultural production.” Discuss.
  6. “There seems to be little doubt that the complete ‘trustification’ of the economy, with the relative stability of prices which would follow therefrom, would go a long way toward eliminating business fluctuations.” Discuss.
  7. “Price stability is prima facie evidence of monopoly.” Discuss.

 

E
Labor Economics and Social Reform

  1. Outline and defend what you would advocate as the best national war-time policy in regard to wages, and whatever else you think must be controlled in order to control wages effectively.
  2. What principal, lasting effects do you think the war is likely to have on the American labor movement – union structures, strength, status, and policies? Explain your predictions and the evidence and reasoning on which you base them.
  3. “The current outcry against federal centralization of unemployment insurance, and in favor of ‘states rights’ in this field, is without merit, and a mere device of employer interests to limit the development of unemployment insurance and keep it as innocuous as possible.”
  4. “American labor unions are deluding themselves in blaming only the false propaganda put out against them by unprincipled opponents, for the better anti-union feelings of some millions of middle and lower-middle-class Americans. Real faults of union leadership and policy have done a great deal to cause and justify this public hostility, and the unions in their own interests can and must assuage it by putting their own houses in order.”
    Discuss this, as far as you can, in terms of concrete, illustrative situations and evidence of which you have some knowledge.
  5. “The Marxian theory that all property-incomes, or non-labor incomes, originate in exploitation of labor, is entirely compatible with the ‘marginal productivity’ theory of income distribution.” Explain and discuss.
  6. Outline, and discuss critically, what you regard as the logical, Marxist explanation of the origins and issues of the present war.
  7. What do you think American Labor, in supporting the war-effort, should put first among its “peace aims”, or aims in respect of the post-war settlement? Explain and defend your answer.

 

PART III

  1. “Economics can either explain the quasi-automatic operation of a true free enterprise economy, or devise a blue-print for rational planning in the socialist economy. But in a half-way house like our present society, where both private and public decisions must respond more often to political than to economic facts, economics can neither explain events nor guide public policy.”
  2. “After the last war, the reaction of business and the public against the war-time government controls gave a new lease of life to laissez-faire, with disastrous results; and there is danger that a like relapse will occur at the end of this war.”
  3. “The proper work of the economists, in helping to solve the problems of industry and society, may be said to begin where that of the engineers or technicians ends.”
  4. “If the opportunity for the employment of idle men and idle money is to be found in a free, private enterprise system then, obviously, we must find a way to stimulate new, private enterprises by encouraging the investment of private savings in them.”
  5. “The causes which bring trade barriers into existence and produce centralism in every form of economic activity must be attacked if a real system of free enterprise is to be re-established.”
  6. “To maintain and improve labor’s position economically is the traditional task of the unions. Today, not only the growth but even the existence of the unions has become in large measure a political problem.”
  7. “The last war, in its impact on the American economy, produced war-time overexpansion and post-war depression chiefly in agriculture. This time, it is the industrial sector of our economy which is threatened with that sequence, on a much more disastrous scale.”
  8. “The patriots who denounce, in war-time, all self-interested demands or actions on the part of business, labor, or farm groups, generally do not recognize the fact that rivalry of all interest-groups over distribution of war-time prosperity is inevitable under our profit-system, and cannot be eliminated unless we are willing to replace that system entirely, while the war lasts, with a governmental dictatorship of all economic life as complete is that now practiced in Germany, Japan, and Russia.”

Source:  Duke University. David M. Rubenstein Rare Book & Manuscript Library. Economists’ Papers Archive. Papers of Randall Hinshaw. Box 1, Folder “Schoolwork, 1940s”.

Image Source: John Harvard Statue from the Tichnor Brothers Collection of postcards. Boston Public Library, Print Department.

Categories
Chicago Exam Questions Fields Undergraduate

Chicago. Comprehensive Exams in Economics for B.A., 1941

 

 

One presumes that a departmental comprehensive examination would cover material that would be expected of any student going on to graduate studies in economics.  The comprehensive examination for Harvard economics majors from 1953 has been previously posted as has Swarthmore’s comprehensive examination for 1931.

A few things worth noting:

  • Henry Simons and Paul Douglas were apparently enough at odds with each other’s economics to be unable to come up with a single principles examination in Part I.
  • Both accounting and basic statistics shared equally in the quantitative Part II.
  • Either U.S. or European Economic History was required to be one of the three field examinations in Part III. A student could even take both economic history examinations, so one can say economic history was very much part of the common core for economists-in-training.
  • From today’s perspective it is interesting to find that “transportation” was a field still having equal status with “labor” and “government finance”.

According to a handwritten note attached to the following comprehensive exam was used four times:  Spring 1940, Winter 1941, Autumn 1941, and (with slight correction) Winter 1942.

__________________

PART I

COMPREHENSIVE EXAMINATION FOR THE BACHELOR’S DEGREE IN ECONOMICS

(Start each new subject in a new examination book)

The comprehensive examination in Economics is divided into three parts:

PART I — Time: Approximately 2 ½ hours.

(a) Principles of Economics
(b) Principles of Money and Banking

PART II — Time: Approximately 2 ½ hours.

(a) Elementary Accounting
(b) Statistics

PART III — Time: Approximately 3 hours.

Write on either (a) or (b) and two other subjects. One of these may be the second subject in Economic History.

(a) Economic History of the United States
(b) Economic History of Europe
(c) Labor
(d) Government Finance
(e) Transportation

 

 

PART I

(a) Economic Principles

Write on either examination A or examination B. In view of the difference in reading lists, examination A is offered primarily for those who did their work in Economics 209 with Mr. Douglas, while examination B is for those who had this course with Mr. Simons.

Examination A.
(Answer all questions.)

  1. Describe in some detail why the demand curves for the products of an industry are negatively inclined and give and illustrate the formula for the measurement of elasticity.
    Why, under atomistic competition, is the demand curve for the products of an individual firm of infinite elasticity and indicate by graphs what forces determine equilibrium for the individual firms (a) with no alternation in their number, (b) in the longer run, where the numbers of firms may vary but where there is no change of the scale of the individual plant, (c) in the still longer run when both the numbers and the scale of plants vary.
  2. Discuss and illustrate equilibrium under conditions of “imperfect competition,” showing (a) the role of average and marginal revenue curves, (b) average and marginal cost curves. Discuss both short-run and long-run equilibrium and the light such conclusions throw upon whether competition is or is not desirable, the proper role of the state, etc.
  3. Trace the theory of production, showing the relative effect upon product of changes in the quantities of the three factors of production, i.e., land, labor and capital, and the steps by which the theory of distribution can be derived from the theory of production.

 

Examination B.
(Answer both questions.)

  1. (50 points)
    In an isolated community there are two kinds of land, and only one product, wheat. There are 100 farms of each The labor supply is homogeneous—i.e., all workers are equally efficient. There is private property in land and free contract for labor. Labor services are bought and sold only in units of one laborer per year. The markets for both labor and land (unless otherwise specified) should be assumed to be freely competitive.
    The table below shows the amounts of wheat which can be obtained from onesingle farm of each grade, with different numbers of laborers per year.
Number of Laborers Output on A-grade Farm Output on B-grade Farm
1 1,000 900
2 1,800 1,200
3 2,400 1,400
4 2,900 1,550
5 3,300 1,650

The labor population is 450 — all workers will seek to be fully employed at any wage rate above zero.

a. What will be the wages per man? Explain why.

b. What will be the rent of farms of each grade?

c. Explain how the productivity (product increment) of an A-grade farm may be determined.

d. What would happen to wages and rents if an output tax of 5 per cent were imposed upon the production of wheat?

e. What would happen to wages and rents if a tax of 100 bushels per farm were levied, the tax being payable by owners?

f. Suppose a minimum wage law is passed and enforced, requiring the payment of at least 700 bushels per year for labor. What will be the effect on total employment and on rents?

g. Suppose that workers on the A-grade farms organize into a trade union and enforce a minimum wage of 700 bushels per year on the A-grade farms. What will happen to rents? To numbers of workers employed on A-grade farms? To the wages of workers not employed on A-grade farms?

h. Suppose that workers organize only on the B-grade farms and enforce there a wage of 700 bushels per year. What will happen to rents? To wages on the A-grade farms?

  1. (50 points)
    Indicate the conditions or circumstances under which each of the following relationships is likely to obtain, in the short run if not in the long run, and explain briefly in each case:

    1. Marginal revenue is equal to price.
    2. Price is equal to average expense (total cost per unit) but far in excess of marginal expense.
    3. Marginal expense, for the industry as a whole, fare exceeds marginal expense for the individual firm.
    4. All firms in a highly competitive industry are maintaining outputs at which their average-cost curves are falling (negatively sloped).
    5. All firms in a highly competitive industry are maintaining outputs at which their marginal-expense curves are falling.
    6. The price of a productive service is equal to its product increment times product price.
    7. The price of a productive service is much less than its product increment times product price.
    8. The price of a productive service is much less than its product increment times marginal revenue (for the firm).
    9. The total output of all firms in an industry is such that marginal revenue, for the industry as a whole, is negative.
    10. Marginal expense and average expense are equal but both are far in excess of product price.

 

(b) Principles of Money and Banking

(Answer all parts in questions 1 and 2; if time permits answer question 3.)

  1. (25 points)
    The following statements are to be completed by filling in the blanks with the most nearly correct of the suggested answers:

    1. Excess reserves of the member banks of the Federal Reserve System are currently about _______ million dollars. (100; 1,000; 1,500; 3,500; 18,700)
    2. The Federal Open Market Committee consists of _______ (5; 7; 9; 12;19) members, of which (1; 3; 5; 7; 12) are members of the Board of Governors of the Federal Reserve System and the remainder selected by ____________________ (President of the U.S.; Board of Governors; U.S. Secretary of the Treasury; directors of the Federal Reserve banks).
    3. In recent months holdings of U.S. Government securities (direct and guaranteed) by the Federal Reserve banks have totaled about _______ million dollars (25; 500; 2,500; 6,000).
    4. A member bank in downtown Chicago is at present required to hold with its Federal Reserve Bank an actual net balance equal to _______ (10; 13; 17½; 22¾; 26) per cent of its net demand deposits.
    5. If the U.S. Treasury were to shift its present deposits from member banks to the Federal Reserve banks, excess reserves of member banks would probably _______ (increase; decrease; remain unchanged) and excess reserves of the Federal Reserve banks _______ (increase; decrease; remain unchanged).
    6. The Board of Governors of the Federal Reserve System is authorized to decrease existing reserve requirements for reserve city member banks to a minimum level of _______ (5; 13; 17½; 20; 100) per cent against its net demand deposits.
    7. The total volume of hand-to-hand money in circulation in the U.S. (in the hands of the public and in banks’ vault cash) has recently been approximately _______ (600; 8,000; 10,000; 50,000) million dollars, of which approximately _______ (0; 5; 25; 30) per cent has consisted of gold coin.
    8. In recent years member banks have held approximately _______ (10; 25; 55; 85; 98) per cent of all demand deposits (excluding inter-bank deposits) in all commercial banks of the country.
    9. If the Federal Reserve banks sold their present holdings of U.S. Government securities to the public, excess reserves of banks in the country would probably _______ (increase; decrease; remain unchanged).
    10. In computing its demand deposits subject to legal reserve requirements, a member bank may deduct from its gross demand deposits _______ (U.S. deposits held with it; balances due from other domestic banks except Federal Reserve banks; its vault cash; balances due to other domestic banks).
    11. In giving a correct statement of the quantity theory of money, it is necessary to state among other things the assumption _______ (that wage rates remain constant; that the country is not on a paper monetary standard; that the economy to which it refers is perfectly competitive; that the theory may not be applicable in the short run).
    12. The monetary gold stock of the United States is currently approximately _______ (3.5; 7.0; 22; 25) billion dollars.
    13. Treasury purchases of imported gold will result in the greatest reduction in excessreserves of banks (not including Federal Reserve banks) when the Treasury pays for the gold by _______ (issuing new gold certificates; borrowing funds from the public; borrowing funds from commercial banks; borrowing funds from the Federal Reserve banks).
    14. Time and demand deposits (excluding interbank deposits) in all banks of the United States currently total about _______ (25; 40; 60; 75) billion dollars, of which amount approximately _______ (10; 25; 40; 60; 98) per cent is fully insured by the Federal Deposit Insurance Corporation.
    15. Under present conditions the Federal Reserve banks can most effectively reduce excess reserves of member banks by _______ (raising the discount rates of the Federal Reserve banks; selling their holdings of U.S. Government securities on the open market; raising the legal reserve ratios of member banks to 100%).
  2. (75 points)
    A recent annual report of the Board of Governors of the Federal Reserve System contained the following statement:
    “Under existing conditions the Treasury’s powers to influence member bank reserves outweigh those possessed by the Federal Reserve System.”

    1. State briefly and concisely the powers of the U.S. Treasury to influence member bank reserves; evaluate and explain their importance with reference to:

(1) Increasing member bank excess
(2) Decreasing member bank excess reserves.

    1. If the Treasury were to use certain of its powers, it could increase its cash holdings (without borrowing or taxing) by 10 billion dollars. Assume that it does so today, and that it spends the 10 billion dollars for national defense goods (in addition to the expenditures previously budgeted) during the next two years. Analyze the effects of the spending, including in your analysis statements concerning the effects on:

(1) Employment and national income.
(2) The cash position of the public.
(3) The reserve position of commercial banks.
(4) The powers of the Federal Reserve System to reduce member bank excess reserves.
(5) Relative changes in important groups of prices.

Of what help is the quantity theory of money to you in explaining the price fluctuations of (5)?

  1. (30 points)
    (If time permits)
    Defend your answers to parts e, I, m, and o of question 1.

 

 

PART II

(a) Elementary Accounting

(Answer all questions; plan to spend at least 40 minutes on question 4.)

  1. Debits and Credits
    Directions: Read the data given and select from the “Numbers To Be Used” the appropriate debit and credit to be used. Write the numbers of these accounts in the appropriate column, indicating in each case the kind of account (A-L-P-E-I).

Numbers to be Used

(1) Accounts Payable (10) Notes Payable
(2) Accounts Receivable (11) Notes Receivable
(3) Bad Debts (12) Office Expense
(4) Cash (13) R. Smith, Capital
(5) Furniture and Fixtures (14) Purchases
(6) General Expense (15) Sales
(7) Interest Cost (16) Wages and Salaries
(8) Interest Income (17) Rent Expense
(9) Merchandise Inventory

 

Debit Credit
Sample: A customer pays us cash on account (4) (A) (2) (A)
1. R. Smith invested cash in a mercantile business 1.
2. Paid cash for rent of store building 2.
3. Bought fixtures for cash 3.
4. Bought merchandise on account 4.
5. Bought office supplies for cash 5.
6. Sold merchandise for cash, note, balance on account 6.
7. Gave a trade creditor a note on account 7.
8. Paid a trade creditor cash on account 8.
9. Paid note payable due a creditor, with interest 9.
10. Received cash on account from a customer 10.
11. Received payment of note due from customer, with interest 11.
12. Paid wages and salaries 12.
13. Paid miscellaneous expenses 13.
14. A customer goes bankrupt and pays only a part of his account, the rest being uncollectible 14.
15. Bought merchandise for cash, note, balance on account 15.
16. Traded merchandise for furniture and fixtures 16.

 

  1. The following statements are to be marked by circling “T” if true, or “F” if false. A statement which is in any part incorrect is to be considered false.

T or F. The declaration of cash dividends results in a current liability on the balance sheet.

T or F. For a corporation having only common stock outstanding, the book value of the common stock is equal to the result obtained by dividing the difference between the total assets and the total liabilities by the number of common shares outstanding.

T or F. Customers’ accounts with credit balances should be shown on the balance sheet as current liabilities.

T or F. If the ending raw materials inventory is valued at too low a figure (other data on the statements correct), the cost of goods sold will be too small.

T or F. If depreciation of an asset is overestimated, that asset will be overvalued on the balance sheet.

T or F. A partnership is always automatically dissolved by the death of any one of its members.

T or F. Stock-dividends declared but not yet issued are shown on the balance sheet as current liabilities.

T or F. If all the stockholders of a corporation die, the corporation ceases to exist.

T or F. Holders of cumulative preferred stock have an unconditional right to dividends that are in arrears.

T or F. If the goods in process inventory at the beginning of an accounting period is overstated (other data on the statements correct), the gross profit for that period will be too small.

T or F. A corporation with a $200,000 surplus account could have no difficulty in paying a $100,000 cash dividend to stockholders.

T or F. Patents are written off to factory expense over the period of their economic life which cannot be more than 17 years.

T or F. Capital surplus represents the amount of profits which the stockholders and directors have been willing to leave invested in the business.

T or F. Expenditures which increase the usefulness of an asset, or prolong its life, are capital expenditures.

T or F. The introduction of controlling accounts for expenses makes necessary some change in the form of the journals used by that business.

T or F. Discount on Stock may be correctly shown on the balance sheet as a deferred charge.

T or F. A sinking fund reserve is set up to prevent the use of sinking fund cash for dividend purposes.

T or F. Preferred stock is never entitled to preference in the distribution of assets in liquidation, unless specified in the stock agreement.

T or F. A firm which has incurred a loss for the year may have more cash on hand at the end of the year than it had at the beginning of that year.

T or F. The cost of repairing a second-hand machine, before it is put to use in the factory, should be charged to factory expense.

 

  1. You are given a Statement of Profit and Loss of the Northwestern Manufacturing Company for the year ended December 31, 1940. Profit is shown as $121,380 Upon investigation you find that the accountant had proceeded as follows:
    1. Inventory had been valued at Market, $180,000; Cost was $150,000.
    2. Depreciation had been calculated on new machinery (purchased January 1, 1940) at a 10% rate. The general experience of competitors indicated that the life of the equipment was five years. The cost of the machine under question was $38,000.
    3. Wages due salesman for services rendered, $8000, had been overlooked.
    4. A garage owned by the Company was destroyed by fire. The building had a book value of $30,000. The insurance company had agreed to pay $20,000. The Company had signed a release but no record had been made of the fire or agreement.
    5. Accounts Receivable were valued at Gross, $200,000.
    6. Competitors had found that about 2% of gross accounts were uncollectible. About $1000 in cash discounts applicable to 1940 were expected to be taken.

What changes would you make on the Balance Sheet and the Statement of Profit and Loss for each of the above items?

  1. List the problems associated with the valuation of fixed assets: (a) at the time of acquisition, (b) of changes subsequent to the time of acquisition. Explain the relationship between these problems and cost determination in a manufacturing enterprise. Suggest solutions which the accountant has used in the past and discuss these critically in terms of economic theory.

 

(b) Statistics

(If time permits, answer all questions; note the unequal weighting, however. Plan to spend approximately 30 minutes on question 3.)

  1. (25 points)
    In the space to the left of each of the following statements indicate whether the statement is true (T) or false (F). Do not guess; if you don’t know whether a statement is true or false, don’t market.

_____ a. In a series of positive numbers the algebraic sum of the deviations of the individual items from their arithmetic mean is positive.

_____ b. In a simple linear correlation the slopes of the two elementary regression lines are always the same.
_____ c. Fisher’s Ideal Index Number formula satisfies both the time reversal and factor reversal tests.
_____ d. A moving average of points which lie along a straight line will reproduce the line.
_____ e. The sum of the squared deviations from the median of the frequency distribution is less than the sum of the squared deviations from any other average of the same frequency distribution.
_____ f. In simple linear correlation the two elementary lines of regression are identical if the simple correlation coefficient (r) is plus one and perpendicular to each other if the simple correlation coefficient is -1.
_____ g. The time series of the population of the United States plots is a straight line on semi-log paper; therefore, we may conclude that the population of the United States has grown at a constant relative rate.
_____ h. The simple correlation coefficient (ryx) is the arithmetic mean of the two simple regression coefficients (bxy and byx).
_____ i. In every frequency distribution 68% of the cases lie within plus and minus one standard deviation from the arithmetic mean.
_____ j. If the simple linear correlation coefficient between X and Y is small, it shows that there is very little relationship of any kind between X and Y.
_____ k. The standard error of estimate for the regression of Y on X depends upon the units in which Y is measured.
_____ l. The aggregative price index with base year quantity weights is identical to the arithmetic index of price relatives weighted by values of the base year.
_____ m. The sampling distribution of means of samples (all of the same size) drawn at random from a normal universe is also normal.
_____ n. The product of the individual items of a series of numbers is unchanged if each of the items is replaced by the geometric mean.
_____ o. The ratios-to-trend method of obtaining an index of seasonal variation is valid only if the underlying trend his linear.
_____ p. If the probability of getting a tail in a single toss of a bias coin is 1/4, the probability of getting three heads in three independent tosses of the same coin is 3/4.
_____ q. The sampling distribution of means of samples (all of the same size) drawn at random from a non-normal universe is less normal than the universe itself.
_____ r. The standard deviation of the sampling distribution of means drawn at random depends upon the size of the samples.
_____ s. The simple geometric average of relative prices satisfies the time reversal test.
_____ t. If a frequency distribution is symmetric when plotted on the arithmetic scale, the geometric mean, the median, and the mode will all coincide.
_____ u. If a frequency distribution is symmetric when plotted with a logarithmic scale on the X-axis, it will be skewed when plotted on the arithmetic scale.
_____ v. The harmonic mean of a series of positive numbers is sometimes greater in the geometric mean.
_____ w. The median is less affected than the arithmetic mean by the magnitude of extreme observations.
_____ x. The probability that two independent observations drawn at random from the same normal universe will both deviate by more than one standard deviation from the arithmetic mean of the universe is approximately 0.32 (= 32%).

  1. (35 points)
    State the reasoning behind your answer to the following parts (seven in all) of question 1:

(a or n)
(b, f, or h)
(c or s)
(i)
(j)
(p or x)
(r)

In each case, if you marked the statement true demonstrate its truth; if you marked it false, revise it so that it is correct, and demonstrate that your revision is true. Use mathematics where convenient.

  1. (40 points)
    The ABC Corporation which manufactures and sells over 1,000,000 packages of cigarettes (20 cigarettes per package) per year advertises of that on the average their cigarettes will burn for 15 minutes (per cigarette).
    The XYZ Corporation, making and selling over 2,000,000 packages of cigarettes per year (20 cigarettes per package) asserts that on the average its cigarettes will burn for 16 minutes (per cigarette).
    The Honesty-in-Advertising Association samples each manufacturer’s cigarettes, taking one sample of 145 cigarettes (not packages) of each Corporation’s. The following is a tabulation of their findings:
Maker of Cigarette Mean Burning Time
(in Minutes)
Sample of [sic] Standard Deviation of Burning Time
(in Minutes)
ABC Corporation 14.5 6.0
XYZ Corporation 15.0 4.0

On the basis of the above findings,

a. Do you feel that the claims of each manufacturer are justified?

b. Do you feel that XYZ cigarettes on the average burn longer than ABC cigarettes.In answering these questions make use of whatever relevant logical techniques you have learned. State your reasoning carefully; your reasoning is even more important than your arithmetic.
Note: The square root of 52 is 7.2.

 

 

PART III

Write on either (a) or (b) and two other subjects.
One of these may be the second subject in Economic History. (Approximately 3 hours).

(a) Economic History of the United States

(Answer the first three questions and, if time remains, the fourth.
Answer in outline form so far as possible.)

  1. Briefly describe or explain.

a. colonial indentured servant;
b. growth of slavery in the colonies;
c. coinage act of 1792;
d. rise of steamboats in the Mississippi Valley;
e. tariff of 1833;
f. railroad land grants of 1862-71;
g. transportation act of 1920;
h. War Industries Board;
i. Congress of Industrial Organization;
j. wages and hours act of 1938.

  1. Enumerate the chief causes for:

a. adoption of the public land act of 1820;
b. decline of canals after 1860;
c. decline of the general price level, 1865-1896;
d. shifted to a favorable balance of commodity trade after 1873;
e. restriction of immigration after 1921;
f. distressed condition of agriculture since 1920;
g. demand for a New Deal in 1933.

  1. Compare the chief exports and imports of about 1860 with those of the post-World War period. Carefully explain the chief economic developments responsible for the changes that took place.
  2. Outline and explain the history of the merchant marine, 1789-1940.

 

(b) Economic History of Europe

(Answer two questions.)

  1. Discuss the significance of any two of the following authors for the student of modern European economic history: Buckle, Tawny, Spengler, Clapham.
  2. Compare the role of the state in industrial enterprise in France and England during the seventeenth century. Did the French or the English government do the most for the general welfare of its people by its industrial policies?
  3. Compare the influence of either the railway or the canal upon the economic development of France, England, and Germany.

(c) Labor

(Answer both questions.)

  1. Discuss:

a. the main features of the various state minimum wage laws and the federal Fair Labor Standards Act;
b. the economic theories upon which they are based;
c. the constitutional issues involved.

  1. Discuss the issues involved as regards structure, membership, aims and methods in the following struggles:

a. The A. F. of L. versus the Knights of Labor.
b. The I.W.W. versus the A. F. of L.
c. Shop committees (or so-called employee representation plans or as sometimes termed “company” and “independent” unions) versus so-called “outside” unions.
d. The C.I.O versus the A.F. of L.

 

(d) Government Finance

(Answer all questions.)

  1. (35 points)

Mark each of the following propositions “True” or “False” and explain briefly (on separate paper):
The exemption, under federal personal-income tax, of interest on the obligations of state and local governments
_____ a. Involves a kind of federal subsidy or grant which is not commendable in terms of the basis on which the different states share relatively.
_____ b. Probably involve serious inequity as among large income receivers of similar income circumstances.
_____ c. Lowers the rate of interest which state and local governments must pay on their new borrowings.
_____ d. Probably serves to retard or delay recovery from severe depressions.
_____ e. Imposes indirectly a significant burden upon persons of small income in their capacity as savers.

  1. (25 points)
    It is often argued that income taxes, while having great merit in other respects, are ill-suited for a predominant place in revenue systems because their revenue-yield fluctuates so widely between years of prosperity and depression. Are such wide fluctuations a fault or virtue in a federal tax? Discuss.
  2. (25 points)
    In spite of its excellent cumulative features, the federal gifts tax leaves large opportunities for avoidance of estates tax through the distribution of property by gift. Explain “cumulative features”; and indicate the relevant facts about the law which have to do with the avoidance opportunities.

 

(e) Transportation

(Answer all questions. Note weighting of questions.)

  1. (10 points)
    In the following statements, underline the figure, or concept, that most nearly accords with accuracy.

    1. Operating expenses of a railroad may be expected to vary in accordance with:
      tons of freight carried; passenger-miles; train-Miles; car-mile; miles of track
    2. The standard gauge of American railroads is:
      3 ft. 6 in.; 4 ft.; 4 ft. 8 in.; 5 ft. 2 in.; 5 ft. 5 in.
    3. The average freight traffic density of American railroads is:
      100,000; 500,000; 1,000,000; 1,500,000; 5,000,000; 10,000,000
    4. The Interstate Commerce Commission was given power to prescribe actual railroad rates in:
      1906; 1903; 1887; 1911; 1920
    5. The carrying capacity of ocean ships is customarily expressed by:
      gross registered tons; deadweight tons; net registered tons; displacement tons; cargo tons of 40 cu. ft.
    6. The regulation of the rates of waterway common carriers in interstate commerce was authorized by Congress in:
      1900; 1916; 1920; 1933
  2. (15 points)
    The following diagram represents two railroad roots and 6 stations, the figures indicating the mileage between each pair of stations. The East and West Railroad serves all these points.

Indicate which of the rate situations stated below are departures from the provisions of the 4th Section of the Interstate Commerce Act:

a. A rate of 50¢ on commodity “X” from A to E, and 75¢ from E to B.

b. A rate of 25¢ on commodity “X” from A to B, and 20¢ from A to C.

c. A rate of 40¢ on commodity “X” from A to D, and 60¢ on commodity “Y” from A to C.

d. A rate of 45¢ on commodity “X” from A to C, and 50¢ on the same commodity from C to E.

e. A rate of 75¢ on commodity “X” from A to F via C, and 50¢ from A to F via E on the same commodity.

  1. (10 points)
    Draw up definitions of “common carrier” and “contract carrier” for the purpose of establishing a system of regulation of water carriers in interstate commerce of the United States.
  2. (20 points)
    The following diagram represents the line of a single railroad with 8 stations. The numbers represent the distances between stations:

Suppose that the rate structure on traffic between these points is represented by the 1st and 5th class rates, and commodity rates on furniture, and steel products, such as sheets, bars, rods.

From A
to
All rates are cents per 100 lbs.
1st Class 5th Class Furniture Iron and Steel
B 25 20 10 16
C 31 22 12 20
D 20 19 10 17
E 37 25 13 22
F 48 30 17 29
G 50 33 20 31
H 50 36 20 31

Assume neither water nor highway competition. What departures from principles of rate-making do you detect in this rate structure?

  1. (15 point)
    The Omnibus Transportation Bill which passed in the House of Representatives last Summer, inter alia, contain the following provisions: “In order that the public at large may enjoy the benefit and economy afforded by each type of transportation, the Commission shall permit each type of carrier or carriers to reduce rates so long as such rates maintain a compensatory return to the carrier or carriers after taking into consideration overhead and all other elements entering into the cost to the carrier or carriers for the service rendered…”Should such a provision be finally adopted into the law and seriously enforced by the Commission, what effect presumably would it have on the freight rate structures, and on the distribution of commodities? Why?
  1. (10 point)
    In which of the following cases is a certificate of public convenience and necessity required? Check the affirmative cases.

    1. A railroad desires to refund a maturing issue of bonds.
    2. Two motor highway common carriers wish to consolidate properties and operations.
    3. John Smith wishes to inaugurate a highway service between Chicago and St. Louis. He has a contract with a St. Louis manufacturer to haul enamel ware to Chicago; and this will take all his facilities northbound. But he desires to secure return loads and will haul any traffic that is offered.
    4. A railroad is about to acquire a new Diesel stream-lined train.
    5. A water common carrier, finding operations entirely unprofitable, decides to abandon operations.
  2. (10 points)
    A common carrier subject to the jurisdiction of the Interstate Commerce Commission files a tariff containing new schedules of rates, embodying a number of changes. Which of the following statements most accurately describes the Commission procedure in dealing with the tariff.

    1. The tariff is passed around among the 11 commissioners, each of whom examines it for possible violations of the first four sections, and the 6 Section of the Act. If the majority of prove it, the tariff is accepted.
    2. The Commission refers it to the standing rate committees of the carriers for determination of the lawfulness of the rates contained therein.
    3. The tariff is received by the Terrace Bureau of the Commission, and checked by its rate clerks for conformance to the provisions of the sixth Section of the Act. If conforming thereto, it is accepted and is permitted to become effective.
    4. The tariff is returned to the carriers with the statement, that since the burden of proof rests upon the carriers to justify the new rates, they must prove that the rates are lawful under the Act before the tariff can be allowed to become effective.
  3. (10 points)
    An ocean steamship line quotes a rate of $10 W/M on automobiles, New York to Liverpool. What would be the ocean freight on an automobile so shipped, weighing 4,000 pounds boxed, and measuring 120 in. by 60 in. by 50 in.?

Source:  University of Chicago Archives. Department of Economics, Records. Box 39, Folder 28.

Image Source: Element from the Social Science Research Building. University of Chicago Photographic Archive, apf2-07449, Special Collections Research Center, University of Chicago Library.

Categories
Chicago Fields Suggested Reading Syllabus

Chicago. Industrial Organization Reading List. Stigler, 1973

 

While the asterisks in the following reading list probably indicate the subset of required course readings, the  list in its entirety may be considered George Stigler’s universe of readings relevant for a graduate student intending to take a comprehensive examination in the field of industrial organization at Chicago. An autobiographical note by George Stigler from 1982 is included at the Nobel Prize website.

_________________

READING LIST
INDUSTRIAL ORGANIZATION
George J. Stigler

Spring, 1973
Business 305
Economics 380

  1. The Definition and Empirical Determination of Competition and Monopoly
    1. Analytical literature

A. P. Lerner, “The Concept of Monopoly”, Review of Economic Studies, Vol. 1

F. H. Knight, Risk, Uncertainty and Profit, p. 76 ff.

G. J. Stigler, “Perfect Competition, Historically Contemplated”, Journal of Political Economy, 1957 (reprinted in Essays in the History of Economics).

*G. J. Stigler, The Organization of Industry, Ch. 2, 3, 4.

*G. Rosenbluth, “Measure of Concentration”, in Business Concentration and Price Policy.

E. F. Fama and A. B. Laffer, “The Number of Firms and Competition”, AER, Sept. 1972.

T. Scitovsky, “Economic Theory and Measurement of Concentration”, in Business Concentration and Price Policy.

M. O. Finkelstein and Richard Friedberg, “The Application of an Entropy Theory of Concentration”, Yale Law Review, March 1967.

H. Demsetz, “Why Regulate Utilities?”, Journal of Law and Economics, April 1968.

    1. Statistical studies

*A. C. Harberger, “Monopoly and Resource Allocation”, American Economic Review, May 1954.

G. Rosenbluth, Concentration in Canadian Manufacturing Industries.

R. Evely and I. M. D. Little, Concentration in British Industry, Ch. 1, pp. 104 ff., 160 ff.

P. Pashigian, “Market Concentration in the United States and Great Britain”, Journal of Law and Economics, October 1968.

Ralph C. Nelson, Concentration in the Manufacturing Industries of the United States, Ch. I-IV.

Carl Eis, “The 1919-1930 Merger Movement in American Industry”, Journal of Law and Economics, October 1969.

F. L. Pryor, “An International Comparison of Concentration Ratios,” Review of Economics and Statistics, May 1972.

M. Gort, “Analysis of Stability and Change in Market Shares”, Journal of Political Economy, 1963.

R. W. Kilpatrick, “A Choice Among Alternative Measures of Industrial Concentration”, Review of Economics and Statistics, May 1967.

*Irvin Grossack, “the Concept and Measurement of Permanent Industrial Concentration, Journal of Political Economy, July/Aug 1972.

  1. Determinants of the Firm-Size Structure
    1. The Economies of Scale

J. McConnell, “Corporate Earnings by Size of Firm”. Survey of Current Business, May 1945.

J. Johnston, Statistical Cost Functions, esp. pp. 110 ff., Ch. 6.

J. Haldi and D. Whitcomb, “Economies of Scale in Industrial Plants”, Journal of Political Economy, 1967.

J. S. Bain, Barriers to New Competition.

*J. S. Bain, “Economies of Scale…” in Readings in Industrial Organization and Public Policy.

*G. J. Stigler, The Economies of Scale, The Organization of Industry, Ch. 7.

P. E. Hart, “The Size and Growth of Firms”,Economica, February 1962.

*F. Modigliani, “New Developments on the Oligopoly Front”, Journal of Political Economy, June 1958.

D. Osborne, “The Role of Entry in Oligopoly Theory”, Journal of Political Economy, 1964.

L. Weiss, “The Survivor Technique and the Extent of Suboptimal Capacity”, Journal of Political Economy, June 1964.

T. Saving, “Estimation of Optimum Size of Plant by the Survivor Method”, Quarterly Journal of Economics, Nov. 1961.

L. Telser, Competition, Collusion and Game Theory, Ch. 8.

    1. Mergers

A. S. Dewing, “A Statistical Test of the Success of Consolidations”, Quarterly Journal of Economics, 1921.

G. J. Stigler, “Monopoly and Oligopoly by Merger”, The Organization of Industry, Ch. 8.

F. T. C., Report on Corporate Mergers and Acquisitions.

*J. Markham, “Survey of the Evidence and Findings on Mergers”, in Business Concentration.

J. F. Weston, The Role of Mergers in the Growth of Large Firms.

G. J. Stigler, “The Statistics of Monopoly and Merger”, Journal of Political Economy, 1956.

*Ralph Nelson, Merger Movements in American Industry.

H. G. Manne, “Mergers and the Market for Corporate Control”, Journal of Political Economy, April 1965.

  1. The Effects of Concentration
    1. Collusion

*D. H. MacGregor, Industrial Combination, Part II, Ch. 1.

W. Fellner, Competition Among the Few.

W. Nicholls, Imperfect Competition Within Agricultural Industries, pp. 120-130.

F. Machlup, Economics of Sellers’ Competition, Ch. 13.

*G. J. Stigler, The Organization of Industry, Ch. 5.

W. Nutter, “Duopoly, Oligopoly, and Emerging Competition”, Southern Economic Journal, 1964.

Lester Telser, Competition, Collusion and Game Theory, Ch. 5.

    1. Price Discrimination

N. I. C. B., Public Regulation of Competitive Practices, pp. 63-85.

J. P. Miller, Unfair Competition, Ch. 7-9.

J. Robinson, Economics of Imperfect Competition, Bk. V.

F. Machlup, The Basing Point System.

G. J. Stigler, A Theory of Uniform Delivered Prices, The Organization of Industry, Ch. 14.

    1. Price Rigidity

*G. Means, Industrial Prices and Their Relative Inflexibility.

Sweezy and Stigler, articles on the kinked oligopoly demand curve in American Economic Association, Readings in Price Theory.

A. C. Neal, Industrial Concentration and Price Inflexibility.

*Stigler, Administered Prices and Oligopolistic Inflation, The Organization of Industry, Ch. 19.

*Stigler & Kindahl, The Behavior of Industrial Prices, Ch. 1, 4, 5.

Government Price Statistics (Joint Economic Committee, 1961, also, National Bureau of Economic Research), Staff Papers No. 8 and 9.

R. Selden and C. dePodwin, “Business Pricing Policies and Inflation”, Journal of Political Economy, 1963.

L. Weiss, “Business Pricing Policies and Inflation Reconsidered,” Journal of Political Economy, 1966.

    1. Profits

*J. S. Bain, “Relation of Profit Rate to Industry Concentration”, Quarterly Journal of Economics, August 1951.

N. R. Collins and Lee E. Preston, Concentration and Price-Cost Margins in Manufacturing Industries.

G. J. Stigler, Capital and Rates of Return in Manufacturing Industries, Ch. 3.

Y. Brozen, “The Antitrust Task Force Deconcentration Recommendation”, Journal of Law and Economics, October, 1970.

S. I. Ornstein, “Concentration and Profits”, Journal of Business, Oct. 1972.

  1. Topics in Industry Behavior
    1. Advertising

E. Chamberlin, Theory of Monopolistic Competition, Ch. 6-7.

L. Telser, “Advertising and Cigarettes”, Journal of Political Economy, October 1962.

*N. Kaldor, “Economic Aspects of Advertising”, Review of Economic Studies, 1950.

*L. Telser, “Advertising and Competition”, Journal of Political Economy, December 1964.

*G. J. Stigler, “The Economics of Information,” The Organization of Industry, Ch. 16.

J. Peterman, “The Clorox Case and the Television Rate Structure”, Journal of Law and Economics, Oct. 1968.

W. S. Comanor and T. A. Wilson, “Advertising Market Structure and Performance”, Review of Economics and Statistics, 1967.

Phillip Nelson, “Information and Consumer Behavior,” Journal of Political Economy, April 1970.

    1. The Nature of the Firm and Vertical Integration

*R. Coase, “The Nature of the Firm”, Readings in Price Theory.

H. Demsetz, “The Exchange and Enforcement of Property Rights”, Journal of Law and Economics, October 1964.

A. Smith, Wealth of Nations, Bk. I, Ch. 3.

Marshall, Principles of Economics, Bk. IV, Ch. 10-13.

A. Young, “Increasing Returns and Economic Progress”, Economic Journal, 1928 (and in Clemence’s Readings in Economic Analysis, 2 vols.)

G. J. Stigler, “Division of Labor is Limited by the Extent of the Market,” The Organization of industry, Ch. 12.

*M. Adelman, “Concept and Measurement of Vertical Integration”, in Business Concentration and Price Policy.

M. Gort, Diversification and Integration in American Industry.

    1. Conglomerate Mergers

C. Edwards, “Conglomerate Progress as a Source of Power”, in Business Concentration and Price Policy.

J. Lorie and P. Halpern, “Conglomerates: The Rhetoric and the Evidence”, Journal of law and Economics, April 1970.

FTC, Economic Report on Corporate Mergers (1969).

    1. Schumpeter’s Theory

*Schumpeter, Capitalism, Socialism and Democracy, Ch. 7-8.

E. Mansfield, “Size of Firm, Market Structure, and innovation”, Journal of Political Economy, 1963.

A. Plant, “Economic Theory Concerning Patents for Invention”, Economica, 1934.

*John McGee, “Patent Exploitation”, Journal of Law and Economics, Oct. 1966.

P. Swan, “Market Structure and Technological Progress”, Quarterly Journal of Economics, 1970.

  1. Large Number Industries
    1. Cartels

G. J. Stigler, The Theory of Price, (1966), Ch. 13.

Clair Wilcox, Public Policies Toward Business(3rd) Ch. 30.

C. Edwards, Economic and Political Aspects of International Cartels.

    1. Trade Associations

*Mancur Olson, The Logic of Collective Action, esp. pp. 125-167.

T. N. E. C. Monograph No. 18, Trade Association Survey.

    1. Retailing: Resale Price Maintenance

W. Bowman, “Prerequisites and Effects of Resale Price Maintenance”, University of Chicago Law Journal, 1955.

F. T. C., Resale Price Maintenance.

*L. Telser, “Resale Price Maintenance”, Journal of Law and Economics, October 1960.

B. Yamey, The Economics of Resale Price Maintenance.

  1. Anti-trust Policy
    1. History

J. D. Clark, Federal Trust Policy.

W. H. Taft, The Anti-trust Act and the Supreme Court.

H. B. Thorelli, The Federal Antitrust Policy.

Robert Bork, “Legislative Intent and the Policy of the Sherman Act”, Journal of Law and Economics, October 1966.

R. Posner, “A Statistical Study of Antitrust Enforcement”, Journal of Law and Economics, 1970.

G. Stigler, “The Economic Effects of the Antitrust Laws,” Journal of Law and Economics, Oct. 1966.

    1. Major dissolutions

E. Jones, Trust Problem in the United States, Ch. 18.

Hale, “Trust Dissolution”, Columbia Law Review, 1940.

W. S. Stevens, Industrial Combinations and Trusts, Ch. 14-15.

S. Whitney, Antitrust Policies, 2 vols.

    1. Law of Conspiracy

U. S. v. Trenton Potteries, 273 U.S. 392 (1927).

F. T. C. v. Cement Institute, 68 Sup. Ct. 793 (1948).

Report of Attorney-General’s National Committee on the Anti-trust Laws.

 

Source: University of Chicago Archives. George Stigler Papers, Box 2, Folder “1970’s: course notes + related: Industrial org. + microeconomics”.

Image Source: George Stigler (November 1977). University of Chicago Photographic Archive, apf3-00844, Special Collections Research Center, University of Chicago Library.

 

Categories
Exam Questions Fields Johns Hopkins

Johns Hopkins. Ten comprehensive economics PhD exams, 1965

 

 

This is what a full set (more-or-less) of written comprehensive examinations from a Ph.D. program in economics looks like as of 1965. The archival records from Johns Hopkins University are in pretty good shape for their graduate program. Earlier exams will be transcribed and posted down the road on the installment plan.

Six exams from 1961 are posted here.

____________________

[Handwritten Note:   Christ. FULL SET EXCEPT: 1) Hist of Theory: oral 2) Ops Res: Courses.]

Ph.D. Written Comprehensive Examination in MICROECONOMICS

DEPARTMENT OF POLITICAL ECONOMY
THE JOHNS HOPKINS UNIVERSITY
April 19, 1965

Candidates should answer each question; all questions are assigned equal weight.

  1. “If a price system did not exist, it would be necessary to create one.” Discuss in some detail recent developments in general equilibrium theory (i.e. in the last fifteen years) that have increased and deepened our understanding of the meaning and limitations of this aphorism.
  2. Suppose that a firm, faced with a downward sloping demand curve for its single product, is maximizing its income from sales at an output where total average cost is rising. The firm is then sold on a competitive market to the highest bidder, who—it is assumed—will also maximize his income when he operates the firm.
    1. What considerations will determine the price paid for the firm?
    2. What will be the income of the new owner from operating the firm?
    3. How will the new output compare with the old?
    4. How will the slope of the total average cost curve in its new situation differ from that in the old?
  3. In a letter to Edgeworth, Marshall once wrote: “You know I never apply curves or mathematics to market values. For I don’t think they help much. And market values are, I think, either absolutely abstract or terribly concrete and full of ever-varying (though individually vital) side-issues. Also Ox for market values measures a stock and not a ‘flow’; and I found that if I once got people to use Demand and Supply curves which discussed stocks along this axis of x, they could not easily be kept from introducing the notion of a stock when flow was essential.”
    Discuss the following: (a) the relation of the ideas expressed in this quotation to Marshall’s period analysis of the firm and industry; (b) the Walrasian treatment of the same issues; (c) the current treatments of these issues, as exemplified in good textbooks on price theory; (d) your own opinion of the validity, relevance and usefulness of the distinction which Marshall is making here.
  4. The cost of providing service on a road bridge at a certain crossing of the Chapaqua River is C = dX + e. The consumer demand for the services of the bridge may be expressed as T = f – gX, ignoring differences in types of traffic. The time spent crossing the bridge by the average user is H = a + bX + cX2, and it is assumed that the opportunity cost of the average user is valued at 80 cents per hour.
    What toll should be set on this bridge?
    (C = cost, in dollars; H = time spent on bridge, in hours; T = toll per vehicle, in dollars; X = number of vehicles using bridge; a, b, c, d, e, f, g are constants in the problem.)
  5. “The real content of the equilibrium concept is to be found not so much in the state itself as in the laws of change which it implies.” (John Chipman).
    Would you agree with this characterization? If so, why? If not, why not? Analyse the concepts which are currently used in order to derive laws of change from consideration of equilibrium states.

____________________

Ph.D. Written Comprehensive Examination in MACROECONOMICS

DEPARTMENT OF POLITICAL ECONOMY
THE JOHNS HOPKINS UNIVERSITY
April 20, 1965

[There are 180 points on the exam – allocate your time carefully.]

  1. Choose 4 from the following set of 5 questions. [Each is worth 25 points.]
    1. Suppose you were the Chairman of the Council of Economic Advisers. Outline and defend your policy program for the next year, paying particular attention to the problems of full employment, growth, stability and the balance of payments.
    2. An extremely general form of an investment function may be written as:
      I = f(y, r, k-1, I-1)
      where I = investment, y = income, r = interest rate, k = capital stock.
      Relate this to

      1. the acceleration principle
      2. the Keynes-Hansen stagnation thesis
      3. the Goodwin model of the business cycle
      4. empirical studies of investment behavior
    3. Discuss the present state of empirical knowledge concerning the nature and properties of the consumption function.
    4. Here are two statements by prominent economists:
      1. “That Keynes’ theory of under-employment equilibrium depends on the assumption of rigid wages should be clear to everybody.” G. Haberler, QJE, Nov. 1949, p. 570.
      2. “The numerous remarks throughout the literature that Keynes relied on wage inflexibilities to obtain his results are entirely unsubstantiated.”
        R. Klein, The Keynesian Revolution, 1947, p. 90.

        1. Using a macro model of your choice, present the details of this classical argument.
        2. Either (1) attempt to reconcile these opposing views,
          or (2) pick one side and defend it.
  1. Give two analyses of macroeconomic behavior, (a) for full employment and (b) for unemployment, using the equation MV = Py (or M = kPy) in each case, where M = money stock, V = 1/k = velocity of money, P = price level, y = real income.
  1. Answer all of the following 5 questions (Each is worth 16 points).
    1. “In growth models of the Harrod-Domar type, behavioral shifts toward more saving have a stimulating effect on GNP, yet in a simple Keynesian model, the opposite is true.” Is this contradiction apparent or real?
    2. Attack or defend the following proposition: “Microeconomics and macroeconomics are not two distinct fields, but are one and students should not be asked to study them separately and be examined in them separately.
    3. Briefly summarize the several empirical and theoretical arguments which support the hypothesis that the stock of money willingly held should be a function of the interest rate.
    4. Discuss briefly the “Patinkin controversy” concerning Say’s law and the quantity theory of money.
    5. “Compensatory fiscal policy must have a weaker effect in the short run if consumption depends on ‘permanent income’ than if consumption depends on current income, other things the same.” Discuss.

____________________

Ph.D. Written Comprehensive Examination in AMERICAN ECONOMIC HISTORY

DEPARTMENT OF POLITICAL ECONOMY
THE JOHNS HOPKINS UNIVERSITY
April 22, 1965

Do both questions in Section I and three out of six in Section II.

  1. a. The attached table presents information on an American industry. Discuss the technological change evident in the data. Is the pattern typical of other nineteenth century industries? Does the pattern tend to substantiate Habakkuk’s position?
    1. Choose any open question in American economic history. Discuss the current thought about this problem and propose some feasible approach to the problem which might dispel much of the uncertainty now obvious in the literature.
  2. a. The nineteenth century immigrant was attracted to the United States by the high American standard of living. Discuss.
    1. What were the principle sources of capital for industry and transportation in the pre-Civil War era?
    2. What role did the foreign sector play in American development during the nineteenth century?
    3. Except for short 3-6 year fluctuations, the rate of growth of the United States has remained relatively stable. Discuss.
    4. Discuss and evaluate one development policy followed by the government in the nineteenth century orthe governmental development policies in one area during that period.
    5. A recent proposal suggests that the growth process can be considered in three stages – each characterized by the principle constraint to growth. During the first stage growth is constrained by the insufficiency of skill, during the second by a lack of domestic capital, and during the final stage by considerations of foreign imbalance. Discuss the application of such a theory to the American experience.

Table

Year U.S. Mass. Large Firms
OUTPUT
(Physical Units)
1820 3,053 468
1831 23,046 7,923 1,264
1837 12,632 3,545
1845 17,568 4,843
1849 76,368 29,875 7,221
1855 31,500 9,165
1859 114,825 41,529 10,560
CAPITAL
(Machine Sets)
1820 215 20
1831 1,246 340 52
1837 565 136
1845 817 174
1849 3,527 1,288 254
1855 1,520 331
1859 5,235 1,673 362
EMPLOYEES
1820 153
1831 621 133 12
1837 198 28
1845 207 33
1849 980 287 45
1855 348 54
1859 1,220 385 53

____________________

Ph.D. Written Comprehensive Examination in INTERNATIONAL ECONOMICS

DEPARTMENT OF POLITICAL ECONOMY
THE JOHNS HOPKINS UNIVERSITY
April 23, 1965

3 hours, maximum points 180.

PART I (10 points each). Answer all.

  1. Explain the principle of comparative advantage, and illustrate with a diagram.
  2. Derive the expression giving the equilibrium relationship between the spot exchange rate and the 90-day forward rate in a two country model with interest rates assumed fixed in both countries.
  3. The official U.S. measure of a balance-of-payments surplus or deficit is the “overall balance.” Define the “overall balance” and indicate the principal shortcomings of this measure.
  4. Define “terms of trade” and indicate the significance of the concept.
  5. What is the “transfer problem”?
  6. Derive the expression for the change in national income resulting from an autonomous increase in exports.

PART II Answer 3 out of 5 (40 points each).

  1. Utopia is a small, highly developed country with capital (K) as its only factor of production. Utopia consumes two goods, x1and x2, and its community utility function is given by U=x1x2. Its production function for xis x1= a1K, and its production function for xis x2= a2Kb. (a1, a2, b are all greater than zero). Its endowment of capital is K. Since Utopia is a small country, it takes the world prices of x1and xas given.
    1. For what values of b is it certain that Utopia will produce either all xand no xor all xand no x1?
    2. If b = 1/2, at what world price of xin terms of xwill Utopia produce only x2?
    3. If b = 1/2, at what world price of xin terms of xwill Utopia produce only x1?
  2. It has been argued that a fixed exchange rate system works better the greater the interest elasticity of international capital flows, and that a flexible rate system works better the smaller the interest elasticity. Being careful to define “works better”, discuss the assumptions that might underlie this contention.
  3. “In a purely static analysis it is clear that a country, far from being annoyed, should be delighted when foreign countries “dump” their exports on world markets at low prices. But dumping invariably gives rise to cries of dismay in the country importing the dumped goods, and rightly so. A consideration of the dynamic effects of dumping leads to the conclusion that a protective tariff is justified in this case.” Discuss.
  4. “In a sense, practically all arguments for tariffs are second-best arguments. If it is desired to alter the internal distribution of income, then the “first-best” policy would be to do so by direct taxes and subsidies. If this is not possible (i.e. there is a constraint in tax policy), it may be done by tariff protection. But while this will achieve the aim it will also have side effects which are adverse, and the welfare attained with the tax constraint will not be as high as without it; indeed, welfare could fall. (Corden)
    Discuss the above contention with respect to any possible aim or aims of tariff policy other than income redistribution.
  5. “The verdict, so far, on the various attempts to test the Heckscher-Ohlin trade theory is, quite simply, ‘inconclusive’.” Discuss this with respect to the predictions of the theory and one or more of the tests to which these have been subjected.

____________________

Ph.D. Written Comprehensive Examination in LABOR

DEPARTMENT OF POLITICAL ECONOMY
THE JOHNS HOPKINS UNIVERSITY
April 23, 1965

PART I. (2 hours, 15 minutes)

  1. Why do people travel faster today than they used to?
  2. Some people believe that there has been no technological change in the education “industry”. Others believe that there has been the same rate of technical change in this area as in the rest of the economy. What implication does this have for the question of whether the rate of return on education has changed over time?
  3. G. Lewis maintains that unions have not raised wages in the U.S. by very much. Do you think this is an accurate statement? Are there any circumstances under which unions might have a substantial effect on real wages?
  4. Do you think that there are any circumstances under which the introduction of featherbedding into a collective bargaining agreement would yield a better allocation of resources?

PART II. (45 minutes)

  1. Define, and comment briefly on each of the following terms:
    1. equalizing difference
    2. secondary boycott
    3. structural unemployment
    4. productivity
    5. wage-push inflation
  2. Most Keynesian macro models include the assumption that the supply of labor is infinitely elastic, at least up to full employment, at the existing money wage. Can this macro rigid money wage assumption be reconciled with the micro assumption that only relative prices matter?

____________________

Ph.D. Written Comprehensive Examination in LOCATION

DEPARTMENT OF POLITICAL ECONOMY
THE JOHNS HOPKINS UNIVERSITY
April 24, 1965

Do all three sections.

  1. Choose an unsettled problem in location theory. Discuss and criticize the current state of the literature. How would you proceed in extending the present knowledge concerning this problem?
  2. a. How would you proceed in analyzing the factors influencing the location of a single industry?
    1. Apply your method to some industry.
  3. a. Discuss the effect of external economies on some industry. (You may not use the industry of section II, part (b).)
    or

    1. Discuss and criticize the technique used in measuring the geographical concentration of industry and regional specialization.

____________________

Ph.D. Written Comprehensive Examination in MATHEMATICAL ECONOMICS

DEPARTMENT OF POLITICAL ECONOMY
THE JOHNS HOPKINS UNIVERSITY
April 23, 1965

Answer 4 questions.

  1. The following are well-known equations summarising certain aspects of the behavior of an individual consumer
    \frac{\partial {{x}_{s}}}{\partial {{p}_{r}}}=-{{x}_{r}}\frac{\partial {{x}_{s}}}{\partial M}+{{x}_{rs}}\,\,\,\,\,\,\,\,\left( r,s=1,2,\ldots ,n \right)
    where xj, pjrepresent quantity and prices of the jthcommodity, M represents money income, and  {{x}_{rs}}=\lambda \frac{{{U}_{rs}}}{U}, where U is the consumer’s utility function.

    1. Derive the equations from a standard model of consumer behavior.
    2. Discuss the sign of the term xrr
    3. Give an interpretation of \lambda .
    4. Outline the effects on the equations of replacing U by f(U), where f´(U) > 0 for all U.
  2. Choosing any suitable general equilibrium model which involves both production and consumption, define an equilibrium for the model, set out postulates which are sufficient to guarantee the existence of the equilibrium, and sketch out the role played by each postulate in giving the guarantee of existence.
    What, in your opinion, is the function of proofs of the existence of an equilibrium?
  3. In a linear production model, there are two activities which use three resources in quantities (2, 1, 1) and (1, 3, 1), respectively, per unit activity level. The resources are available in quantities (10, 10, 5.5) at most. Receipts from unit levels of the two activities are (4,3).
    1. Set up the problem of maximizing total receipts in linear programming form.
    2. Write down the dual problem.
    3. The solution is included among activity vectors (4,2), (4.5,1), (3.25, 2.25) and dual vectors (1,0,2) (0,3,1). Identify the infeasible activity vector and the optimal activity vector, giving reasons.
    4. State the maximum prices the firm would be willing to pay for its resources.
    5. Discuss the relationship between dual variables in linear programming theory and prices in economic theory.
  4. Set out the leading properties of square non-negative matrices. Show how these properties are used in economic theory in relation to
    (a) input-out models of Leontief type, and
    (b) stability analysis of multi-market equilibrium.
  5. Carefully describe the Von Neumann growth model, interpreting its chief results and relating them to other, not necessarily linear, multisector growth models.

____________________

Ph.D. Written Comprehensive Examination in MONEY

DEPARTMENT OF POLITICAL ECONOMY
THE JOHNS HOPKINS UNIVERSITY
April 23, 1965

PART I (60 points). Indicate whether each of the following statements is true, false, or uncertain, and explain your answer.

  1. If the quantity theory holds, the level of real output and the price level are independent of the supply and/or mix of non-money financial assets.
  2. At full employment the equation of exchange goes from an irrefutable identity to a useful price-level empirical relationship.
  3. The rate of interest is purely a real phenomenon if money is neutral.
  4. The Treasury should lengthen the average maturity of the public debt when long-term interest rates are low.
  5. The demand for money is a function of the real rate of interest rather than the money rate of interest.
  6. If a commercial bank raises new capital by a common stock issue, its reserves will rise permitting a multiple expansion in the money supply.

PART II (60 points). Listed below are four proposals for new legislation. For each proposal, discuss the probable economic effects were the proposal to be enacted and indicate your views on the desirability of the proposal. (Consider each proposal separately; i.e., as you discuss one proposal assume that none of the others has been enacted.)

  1. Require the Federal Reserve to support the prices of Government securities whenever their yields reach 4 ¼ percent.
  2. Allow commercial banks to pay any interest they deem proper on demand and time deposits.
  3. Require the Federal Reserve to produce a money supply which is as close as is practicable to a 3 percent annual rate growth path.
  4. Provide that members of the Board of Governors of the Federal Reserve System may be appointed to, or removed from, office at any time by the President of the United States.

 

PART III. (60 points). Answer two.

[Handwritten note: THIS PAGE WAS NOT GIVEN. (i.e. Part III)]

  1. “Economists are frequently skeptical about the effects of price flexibility in goods markets because they feel that price declines will lead to expectations of further price declines resulting in a reduction in consumption. If expectations as to future prices of long-term bonds are determined in a similar way, the liquidity trap will not exist and, in fact, the economy will be very sensitive to changes in monetary policy.” Discuss.
  2. Today, monetary policy is generally considered to be useful for dealing with economy-wide stabilization problems but not with regional stabilization problems; overheated regions and depressed regions must be treated with different medicines. But the Federal Reserve System consists of twelve regional banks and its founders thought a regional monetary policy was possible. What economic conditions would have to prevail before a regional monetary policy would be possible?
  3. Outline the major aspects of the Gurley-Shaw hypothesis, and discuss its merits. What empirical observations would lead you to accept or reject the hypothesis? (Is the fact relevant that in recent months the rate of growth of money supply plus time deposits has remained unchanged while the rate of growth of money supply alone has declined substantially?)

    ____________________

Ph.D. Written Comprehensive Examination in PUBLIC FINANCE

DEPARTMENT OF POLITICAL ECONOMY
THE JOHNS HOPKINS UNIVERSITY
April 24, 1965

PART I (40 min.)

For each of the following indicate whether the statement is true, false, or uncertain, and then explain your reasoning. (Statements in parentheses may be taken as correct.)

  1. (In a suburban Los Angeles owner-occupied housing development in a recent year, real estate tax payment as a fraction of annual family income was higher for lower-income families than for higher-income families.) The foregoing fact shows that the residential real estate tax is a regressive tax.
  2. There is no advantage or disadvantage to the investor in holding tax-exempt municipal bonds because market demands will have adjusted so as to equate the after-tax yields on the two.
  3. (Available empirical evidence indicates that the after-tax rate of return on equity in the corporate sector has changed very little since the late 1920’s despite the fact that the corporate income tax rate has increased from 11% to over 50%.) From this evidence one can conclude that the effect of the tax has been to raise corporate product prices or to lower the prices of corporate-hired factors of production, or some combination of these two, so that the tax has been simply shifted away from corporate profits with no change in resource allocation.
  4. Marginal cost pricing should be employed for all public utility undertakings.

 

PART II (70 min.) Answer any 2 of the following:

  1. Discuss briefly the issues raised by the proposal of Mr. Heller (former chairman of the Council of Economic Advisers) to transfer certain Federal tax receipts to the states.
  2. Discuss briefly the consequences of Representative Patman’s proposal to finance the Great Society by a loan of $25 billion from the Federal Reserve to the Treasury at an interest rate of ¼ of 1% per year.
  3. Discuss the likely impact of the current proposal to reduce excise taxes upon
    1. The level of aggregate demand
    2. The degree of automatic stabilization of the economy
    3. The distribution of income if taxes on luxury items are eliminated.
  4. Compare, contrast, and evaluate the effects and merits of the following programs to stimulate investment:
    1. Militant easy money programs
    2. Tax inducements such as
      1. Accelerated depreciation
      2. Investment tax credits
      3. Shifts to consumption-expenditure taxation away from income taxation.

 

PART III (70 min.) Answer any 2 of the following:

  1. Indicate the role of government in promoting economic growth in the following models:
    1. Neo-classical models of economic growth à la Solow.
    2. Fixed-proportions models à la Domar.
    3. An economy where government stabilization policy is required to maintain full employment.
  2. Define a public good; a social want. What special problems do they pose to a government which is interested in both resource allocation and optimal income distribution?
  3. Two countries decide to join in an economic union. They wish to have free trade and optimal resource allocation within the union.
    1. One country has a tax system relying solely on a value-added tax and the other one has a proportional income tax. Would an adjustment to exporters or importers be necessary for these taxes?
    2. Suppose the taxes were a value-added tax and a proportional tax on consumption. Would your answer be the same?
  4. How could the government use fiscal measures to improve resource use in transportation?

____________________

Ph.D. Written Comprehensive Examination in ECONOMETRICS

DEPARTMENT OF POLITICAL ECONOMY
THE JOHNS HOPKINS UNIVERSITY
April 24, 1965

Part I

Define or explain briefly:

  1. multicollinearity
  2. consistent estimator
  3. asymptotically efficient estimator
  4. asymptotic variance of an estimator
  5. asymptotically normally distributed estimator
  6. best linear unbiased estimator
  7. Monte Carlo study
  8. specification bias
  9. specification error
  10. autoregressive disturbances

 

Part II (answer two out of three)

  1. Write an essay on the identification problem in which you indicate (a) the definition of “identification”, (b) why the problem arose in the economic literature rather than in the broader statistical literature, (c) the relation of identification to estimation, and (d) some of the important theoretical results concerning identification criteria. In (d), be careful to state carefully the assumptions on which the results rest.
  2. Describe the two-stage least squares estimator in words. Then write down a mathematical expression for it, defining your symbols.
  3. Discuss some of the principal difficulties in estimation that are sometimes encountered when disturbances are serially dependent, and indicate under what circumstances each arises or does not arise. Indicate also what techniques (if any) are available to deal with each difficulty, and how they deal with the difficulty.

 

Part III (answer two out of four)

  1. Suppose that current consumption cwere a linear function of current expected income yet, with an additive disturbance uthat is serially independent with mean zero and constant variance. Suppose also that current expected income is a weighted average of current actual income yand lagged expected income ye,t-1, with constant weights and with an additive disturbance vthat is independent of uand serially independent with mean zero and constant variance. Suppose also that income is exogenous. What problems would be encountered in the attempt to obtain consistent and asymptotically efficient estimators of the slope and intercept of the consumption function described? What information and techniques would enable you to solve them? Explain.
  2. Discuss the effects of multicollinearity upon the estimation of parameters by ordinary least squares. In your discussion assume that the distribution of disturbances is spherical and that the regressor observation matrix X is fixed in repeated samples.
  3. Specify the necessary conditions under which the method of instrumental variables provides consistent estimators in circumstances where ordinary least squares fails to do so.
  4. If a regressor is correlated with the disturbance then classical least squares estimation will result in a biased estimator of the influence of variations in the regressor on variations in the regressand. What can you say about the direction and extent of this bias?

 

Source:  Johns Hopkins University, Eisenhower Library, Ferdinand Hamburger, Jr. Archives. Department of Political Economy, Series 6, Box 1, Folder “Comprehensive Exams for Ph.D. in Political Economy 1947-1965”.

Image Source: From the Johns Hopkins University seal on the cover of the 1966 yearbook Hullabaloo.